Nothing Special   »   [go: up one dir, main page]

PM Shri KV Gachibowli Science Class X 10 Sample Papers For Practice

Download as pdf or txt
Download as pdf or txt
You are on page 1of 78

Science 10 Sample paperS

FOR CLASS X
PREPARED BY: M. S. KUMAR SWAMY, TGT(MATHS)
PM SHRI KENDRIYA VIDYALAYA GACHIBOWLI, GPRA CAMPUS, HYD-32
SAMPLE PAPER TEST 01 FOR BOARD EXAM 2024

SUBJECT: SCIENCE MAX. MARKS: 80


CLASS : X DURATION: 3 HRS
General Instruction:
1. This Question Paper has 5 Sections A-E.
2. Section A has 20 MCQs carrying 1 mark each.
3. Section B has 6 questions carrying 02 marks each.
4. Section C has 7 questions carrying 03 marks each.
5. Section D has 3 questions carrying 05 marks each.
6. Section E has 3 case based integrated units of assessment (04 marks each) with sub-parts of the
values of 1, 1 and 2 marks each respectively.
7. All Questions are compulsory. However, an internal choice in 2 Qs of 5 marks, 2 Qs of 3 marks
and 2 Questions of 2 marks has been provided. An internal choice has been provided in the
2marks questions of Section E
8. Draw neat figures wherever required. Take π =22/7 wherever required if not stated.
SECTION – A
Questions 1 to 20 carry 1 mark each.
1. The image shows the ray diagram of a defected eye.

Which option shows the correction of the defect of the eye?

(a)

(b)

(c)

(d) None of these

2. The image of an object placed in front of a convex mirror is formed at


(a) the object itself
(b) twice the distance of the object in front of the mirror
(c) half the distance of the object in front of the mirror
(d) behind the mirror

Prepared by: M. S. KumarSwamy, TGT(Maths) Page - 1 -


3. Sodium hydrogen carbonate when added to acetic acid evolves a gas.
Which of the following statements are true about the gas evolved?
(i) It turns lime water milky
(ii) It extinguishes a burning splinter
(iii) It dissolves in a solution of sodium hydroxide
(iv) It has a pungent odour
(a) (i) and (ii) (b) (i), (ii) and (iii) (c) (ii), (iii) and (iv) (d) (i) and (iv)

4. Common salt besides being used in kitchen can also be used as the raw material for making
(I) washing soda (II) bleaching powder (III) baking soda (IV) slaked lime
(a) (I) and (II) (b) (I), (II) and (IV) (c) (I) and (III) (d) (I), (III) and (IV)

5. What happens when calcium is treated with water?


(I) It does not react with water
(II) It reacts violently with water
(III) It reacts less violently with water
(IV) Bubbles of hydrogen gas formed stick to the surface of calcium
(a) (I) and (IV) (b) (II) and (III) (c) (I) and (II) (d) (III) and (IV)
6. Choose the correct path of urine in our body
(a) kidney → ureter → urethra → urinary bladder
(b) kidney → urinary bladder → urethra → ureter
(c) kidney → ureters → urinary bladder → urethra
(d) urinary bladder → kidney → ureter → urethra
7. In the reaction of iron with copper sulphate solution: CuSO4 + Fe → Cu + FeSO4
Which option in the given table correctly represents the substance oxidised and the reducing
agent?
OPTION Substance Oxidized Reducing Agent
(a) Fe Fe
(b) Fe FeSO4
(c) Cu Fe
(d) CuSO4 Fe
8. Given below is a diagrammatic representation of a process taking place in the human body.

Prepared by: M. S. KumarSwamy, TGT(Maths) Page - 2 -


In which of these regions/organs could it be occurring?
(i) lungs (ii) heart (iii) brain
(a) only in (i) (b) only in (ii) (c) only in (i) and (ii) (d) in all - (i), (ii) and (iii)

9. In the given food chain, suppose the amount of energy at the fourth trophic level is 5 kJ, what
will be the energy available at the producer level?
Grass → Grasshopper → Frog → Snake → Hawk
(a) 5 k J (b) 50 k J (c) 500 k J (d) 5000 k J

10. The percentage of solar radiation absorbed by all the green plants for the process of
photosynthesis is about
(a) 1 % (b) 5 % (c) 8 % (d) 10 %

11. A beam of light is incident through the holes on side A and emerges out of the hole on the other
face of the box as shown in the figure. Which of the following could be inside the box?
(a) Concave lens (b) Rectangular glass slab
(c) Prism (d) Convex lens

12. In peas, a pure tall plant (TT) is crossed with a short plant (tt). The ratio of pure tall plants to
short plants in F2 is
(a) 1 : 3 (b) 3 : 1 (c) 1 : 1 (d) 2 : 1

Alkaline KMnO 4
13. CH3 — CH2 — OH  Heat
CH3 — COOH
In the above given reaction, alkaline KMnO4 acts as
(a) reducing agent (b) oxidising agent (c) catalyst (d) dehydrating agent
14. A conducting wire carries 1021 electrons in 4 minutes. What is the current flowing through the
wire?
(a) 40 A (b) 7 A (c) 4 A (d) 0.7 A
15. An electric toaster has a power rating of 200 W. It operates for 1 hour in the morning and 1 hour
in the evening. How much does it cost to operate the toaster for 10 days at Rs. 5 per kW h?
(a) Rs. 20 (b) Rs. 400 (c) Rs. 5000 (d) Rs. 10000
16. Choose the incorrect statement from the following regarding magnetic lines of force
(a) The direction of magnetic field at a point is taken to be the direction in which the north pole
of a magnetic compass needle will point
(b) Magnetic field lines are closed curves
(c) If magnetic field lines are parallel and equidistant, they represent zero field strength
(d) Relative strength of magnetic field is shown by the degree of closeness of the field lines
DIRECTION: In the question number 17 and 20, a statement of Assertion (A) is followed by a
statement of Reason (R).
Choose the correct option
(a) Both assertion (A) and reason (R) are true and reason (R) is the correct explanation of
assertion (A)
(b) Both assertion (A) and reason (R) are true and reason (R) is not the correct explanation of
assertion (A)
(c) Assertion (A) is true but reason (R) is false.
(d) Assertion (A) is false but reason (R) is true.
Prepared by: M. S. KumarSwamy, TGT(Maths) Page - 3 -
17. Assertion(A): The sex of a child in human beings will be determined by the type of
chromosome he/she inherits from the father.
Reason(R): A child who inherits ‘X’ chromosome from his father would be a girl (XX), while a
child who inherits a ‘Y’ chromosome from the father would be a boy (XY).

18. Assertion (A): In electrolysis of water, the volume of hydrogen liberated is twice the volume of
oxygen formed.
Reason (R): Water (H2O) has hydrogen and oxygen in the ratio of 1 : 2 by volume.

19. Assertion(A): Large concave mirrors are used to concentrate sunlight to produce heat in solar
cookers.
Reason(R): Concave mirror converges the light rays falling on it to a point.

20. Assertion (A): Myopia is the defect of the eye in which only nearer objects are seen by the eye.
Reason (R): The eye ball is elongated.

SECTION – B
Questions 21 to 26 carry 2 marks each.

21. What is the role of saliva in the digestion of food?

22. Give the pair of contrasting traits of the following characters in pea plant and mention which is
dominant and recessive.
(i) yellow seed (ii) round seed

23. (i) Write the number of covalent bonds in the molecule of propane, C3H8.
(ii) Which element exhibits the property of catenation to maximum extent and why?
OR
Catenation is the ability of an atom to form bonds with other atoms of the same element. It is
exhibited by both carbon and silicon. Compare the ability of catenation of the two elements.
Give reasons.

24. Which among the following are physical or chemical changes?


(i) Evaporation of petrol
(ii) Burning of Liquefied Petroleum Gas (LPG)
(iii) Heating of an iron rod to red hot
(iv) Curdling of milk
(v) Sublimation of solid ammonium chloride

25. An electric oven of 2 kW power rating is operated in a domestic electrical circuit of 220 V that
has a current rating of 5 A. What result do you expect? Explain.
OR
Two lamps, one rated 100 W; 220 V, and the other 60 W; 220 V, are connected in parallel to
electric mains supply. Find the current drawn by two bulbs from the line, if the supply voltage is
220 V.

26. What is translocation? Why is it essential for plants?

SECTION – C
Questions 27 to 33 carry 3 marks each.
27. Size of image of an object formed by a mirror having a focal length of 20 cm, is observed to be
reduced to 1/3rd of its size. At what distance the object has been placed from the mirror? What
is the nature of the image and the mirror?

Prepared by: M. S. KumarSwamy, TGT(Maths) Page - 4 -


28. (i) Why are budding, fragmentation and regeneration all considered as asexual types of
reproduction?
(ii) With neat diagrams explain the process of regeneration in Planaria.
OR
Reproduction is one of the most important characteristics of living beings. Give three reasons in
support of the statement.

29. What are reflex actions? Give two examples. Explain a reflex arc.

30. Read the following information:


I. Resistivity of copper is lower than that of aluminium which in turn is lower than that of
constantan.
II. Six wires labelled as A, B, C, D, E, F have been designed as per the following parameters:
Wire Length Diameter Material Resistance
A l 2d Aluminium R1
B 2l d/2 Constantan R2
C 3l d/2 Constantan R3
D l/2 3d Copper R4
E 2l 2d Aluminium R5
F l/2 4d Copper R6

Answer the following questions using the above data:


(i) Which of the wires has maximum resistance and why?
(ii) Which of the wires has minimum resistance and why?
(iii) Arrange R1, R3 and R5 in ascending order of their values. Justify your answer.

31. The flow of energy between various components of the environment has been extensively
studied. Give an outline of the findings.

32. In an industrial process used for the manufacture of sodium hydroxide, a gas ‘A’ is formed as a
by-product. The gas ‘A’ reacts with lime water to give a compound ‘B’ which is used as a
bleaching agent in the chemical industry. Identify ‘A’ and ‘B’. Also give the chemical equations
of the reactions involved.
OR
A metal A, which is used in thermite process, when heated with oxygen gives an oxide B, which
is amphoteric in nature. Identify A and B. Write down the reactions of oxide B with HCl and
NaOH.

33. A silver article generally turns black when kept in the open for a few days. The article when
rubbed with toothpaste again starts shining.
(i) Why do silver articles turn black when kept in the open for a few days? Name the
phenomenon involved.
(ii) Name the black substance formed and give its chemical formula.

SECTION – D
Questions 34 to 36 carry 5 marks each.

34. Define pollination. Explain the different types of pollination. List two agents of pollination.
How does suitable pollination lead to fertilization?
OR
(i) Write the reaction that occurs when glucose breaks down anaerobically in yeast.
(ii) Write the mechanism by which fishes breathe in water.
(iii) Name the balloon like structures present in lungs. List its two functions.
(iv) Name the respiratory pigment and write its role in human beings.

Prepared by: M. S. KumarSwamy, TGT(Maths) Page - 5 -


35. Draw the pattern of the field lines of the magnetic field around a current carrying straight
conductor passing through and held perpendicular to a horizontal cardboard. State right-hand
thumb rule and explain how this rule is useful to determine the direction of the magnetic field in
the above case, if the direction of current in the conductor is vertically downwards.
OR
With the help of a labelled diagram, explain the distribution of magnetic field due to a current
through a circular loop. Why is it that if a current carrying coil has n turns the field produced at
any point is n times as large as that produced by a single turn?

36. Why are certain compounds called hydrocarbons? Write the general formula for homologous
series of alkanes, alkenes and alkynes and also draw the structure of the first member of each
series. Write the name of the reaction that converts alkenes into alkanes and also write a
chemical equation to show the necessary conditions for the reaction to occur.

SECTION – E(Case Study Based Questions)


Questions 37 to 39 carry 4 marks each.
37. Case Study – 1
Humans use sexual mode of reproduction. But the actual transfer of germ cells between two
people needs special organs for the sexual act. In mammals such as humans, the baby is carried
in the mother’s body for a long period and is breastfed later on. The female reproductive organs
and breasts will need to mature to accommodate these possibilities. Hence some specialised
systems are involved in the process of sexual reproduction. The given graph shows the
hormonal changes during a normal menstrual cycle.

(i) What would be a likely consequence if the hormone represented by graph Q is lacking in an
adult female?
(ii) What is funeral of unfertilised egg?
(iii) After the beginning of menstrual cycle, at which day progesterone reaches its peak? Give
reason.
OR
(iii) (a) Name two simple organisms having the ability of regeneration.
(b) What is the role of the seminal vesicles and the prostate gland?

38. Case Study - 2


The image formed by a convex lens depends on the position of the object in front of the lens.
When the object is placed anywhere between focus and infinity, the image formed by convex
lens is real and inverted. The image is not obtained on the screen when the object is placed
between focus and the lens.
The distance between the optical centre O of the convex lens and the focus point F1 or F2 is its
focal length.

Prepared by: M. S. KumarSwamy, TGT(Maths) Page - 6 -


When the object shifts from –∞ to F1, the image moves from F2 to +∞.

When the object shifts from F1 to O, the image moves from –∞ to O.

A student did an experiment with a convex lens. He put an object at different distances from the
lens. In each case he measured the distance of the image from the lens. The results were
recorded in the following table.
Object distance (in cm) 25 30 40 60 120
Image distance (in cm) 100 24 60 30 40
Unfortunately, his results are written in the wrong order.
(i) Arrange the image distance in the correct order (in cm).
(ii) Which of the object distances gives the biggest image? Give reason.
(iii) Find the focal length of this lens.
OR
(iii) What is the minimum distance between an object and its real image formed by a convex
lens? Where should an object be placed to get a virtual image by convex lens?

39. Case Study – 3


For an internal combustion engine to move a vehicle down the road, it must convert the energy
stored in the fuel into mechanical energy to drive the wheels. In your car, the distributor and
battery provide this starting energy by creating an electrical “spark”, which helps in combustion
of fuels like gasoline. Below is the reaction depicting complete combustion of gasoline in full
supply of air:
2C8H18 (l) + 25 O2 (g) → 16 X + 18 Y
(i) Identify the types of chemical reaction occurring during the combustion of fuels? Name the
product ‘X’ and ‘Y’.
(ii) ‘Although nitrogen is the most abundant gas in the atmosphere, it does not take part in
combustion’. Justify the statement.
(iii) ‘A student while walking on the road observed that a cloud of black smoke belched out
from the exhaust stack of moving trucks on the road.’ Give reason.
OR
(iii) Write the balanced chemical equations for the following reaction and identify the type of
reaction.
Ethene is burnt in the presence of oxygen to form carbon dioxide, water and releases heat and
light.

Prepared by: M. S. KumarSwamy, TGT(Maths) Page - 7 -


PM SHRI KENDRIYA VIDYALAYA GACHIBOWLI, GPRA CAMPUS, HYD-32
SAMPLE PAPER TEST 02 FOR BOARD EXAM 2024

SUBJECT: SCIENCE MAX. MARKS: 80


CLASS : X DURATION: 3 HRS
General Instruction:
1. This Question Paper has 5 Sections A-E.
2. Section A has 20 MCQs carrying 1 mark each.
3. Section B has 5 questions carrying 02 marks each.
4. Section C has 6 questions carrying 03 marks each.
5. Section D has 4 questions carrying 05 marks each.
6. Section E has 3 case based integrated units of assessment (04 marks each) with sub-parts of the
values of 1, 1 and 2 marks each respectively.
7. All Questions are compulsory. However, an internal choice in 2 Qs of 5 marks, 2 Qs of 3 marks
and 2 Questions of 2 marks has been provided. An internal choice has been provided in the
2marks questions of Section E
8. Draw neat figures wherever required. Take π =22/7 wherever required if not stated.
SECTION – A
Questions 1 to 20 carry 1 mark each.
1. Match the labelled parts of the given figure with the correct option.

A B C D E
(a) Fallopian tube Oviduct Uterus Cervix Vagina
(b) Oviduct Vas deferens Ovary Vagina Cervix
(c) Ovary Oviduct Uterus Cervix Vagina
(d) Ovary Fallopian tube Uterus Vagina Cervix
2. When a big fish eats a small fish, which eats water fleas supported by phytoplankton, the water
fleas are
(a) primary consumers (b) secondary consumers (c) top consumer (d) producers.
3. Given a point source of light, which of the following can produce a parallel beam of light?
(a) Concave lens (b) Two plane mirrors inclined at 90° to each other
(c) Convex mirror (d) Concave mirror
4. The trachea is prevented from collapsing by
(a) complete cartilaginous rings (b) incomplete cartilaginous rings
(c) bony rings (d) chitinous rings.
5. Refer to the given figures (i), (ii) and (iii).

Prepared by: M. S. KumarSwamy, TGT(Maths) Page - 1 -


Which of these appears more accurate?
(a) (i) and (iii) (b) (ii) only (c) (i) only (d) (ii) and (iii)

6. The magnetic field produced due to a circular wire at its centre is


(a) at 45° to the plane of the wire (b) at 60° to the plane of the wire
(c) in the plane of the wire. (d) perpendicular to the plane of the wire

7. Which among the following is(are) double displacement reaction(s)?


(i) Pb + CuCl2 → PbCl2 + Cu
(ii) Na2SO4 + BaCl2 → BaSO4 + 2NaC1
(iii) C + O2 → CO2
(iv) CH4 + 2O2 → CO2 + 2H2O
(a) (i) and (iv) only (b) (ii) only (c) (i) and (ii) only (d) (iii) and (iv) only

8. The given diagram represents a __________ reaction.

(a) photodecomposition (b) electrolysis (c) displacement (d) thermal decomposition

9. A magnetic field exerts no force on


(a) a stationary electric charge (b) a magnet
(c) an electric charge moving perpendicular to its direction (d) an unmagnetised iron bar

10. Which of the following is true about point P?


(a) Current is flowing in direction 4.
(b) Electron is flowing in direction 4.
(c) Electrons is flowing in either direction 1 or direction 3.
(d) Electron is flowing in direction 2.

11. An object 20 cm from a spherical mirror give rise to a


virtual image 15 cm behind the mirror. The type of the
mirror and its focal length is
(a) concave, 8.5 cm (b) convex, 30 cm (c) concave, 60 cm (d) convex, 60 cm

12. What are (X), (Y) and (Z) in the given figure?

Prepared by: M. S. KumarSwamy, TGT(Maths) Page - 2 -


X Y Z
(a) H2 Cl2 NaOH
(b) NaOH H2 Cl2
(c) NaOH Cl2 H2
(d) Cl2 H2 NaOH

13. Observe the given figure and answer the question that follows:

Bulb glows when _________ wire is placed between the terminals A and B.
(i) Copper (ii) Aluminium (iii) Iron (iv) Silver
(a) (i) and (iii) (b) (i), (ii) and (iii) (c) Only (iv) (d) (i), (ii), (iii) and (iv)

14. Select the correct statement(s) regarding the given allotropes of carbon.

(i) A crystal of (II) is a giant molecule consisting of only carbon atoms.


(ii) The density of (II) is much lower than that of (I).
(iii) Allotrope (III) is the softest one.
(a) (i) only (b) (i) and (iii) only (c) (ii) only (d) (i), (ii) and (iii)
15. In the given series of reactions, what are Y and Z respectively?
 H 2O ,  CO2 10 H 2O
NaCl + H2O + CO2 + NH3 → X + Y; X  
Z  Q
(a) NaHCO3, NaOCl2 (b) NH4Cl, Na2CO3 (c) Na2CO3, NH4Cl (d) Na2CO3, NaHCO3
16. pH of different solutions are given in the table below:
Solution pH
P 2.2-2.4
Q 13.8-14.0
R 6.5-7.5
S 8.0-9.0
Arrange these solutions in the increasing order of OH ion concentration.
(a) S<R<Q<P (b) P<R<S<Q (c) R<S<Q<P (d) Q<S<R<P

DIRECTION: In the question number 17 and 20, a statement of Assertion (A) is followed by a
statement of Reason (R).
Choose the correct option
(a) Both assertion (A) and reason (R) are true and reason (R) is the correct explanation of
assertion (A)
(b) Both assertion (A) and reason (R) are true and reason (R) is not the correct explanation of
assertion (A)

Prepared by: M. S. KumarSwamy, TGT(Maths) Page - 3 -


(c) Assertion (A) is true but reason (R) is false.
(d) Assertion (A) is false but reason (R) is true.
17. Assertion (A): A current carrying wire should be charged.
Reason (R): The current in a wire is due to flow of free electrons in a definite direction.

18. Assertion (A): 2, 4-Dimethylhept-5-ene is

Reason (R): The double bond gets preference to the alkyl group.

19. Assertion (A): Capillaries are the thinnest blood vessels.


Reason (R): Capillaries connect the branches of arteries and veins.

20. Assertion (A): Monohybrid cross deals with inheritance of one pair of contrasting characters.
Reason (R): Dihybrid cross deals with inheritance of two pairs of contrasting characters.

SECTION – B
Questions 21 to 26 carry 2 marks each.
21. (a) Which of the following flowers will have higher possibility of self-pollination?
Mustard, Papaya, Watermelon, Hibiscus
(b) List the two reproductive parts of a bisexual flower.

22. What is variation? List two main reasons that may lead to variation in a population.
OR
Is it possible that a trait is inherited but not expressed in an organism? Explain on the basis of
Mendel experiment.

23. Explain the formation of a rainbow in the sky.


24. A rectangular coil ABCD is placed between the pole pieces of a horse-shoe magnet as shown in
figure.

(i) What is the direction of force on each arm?


(ii) What is the effect of the forces on the coil?
OR
A current carrying conductor is placed in a magnetic field now answer the following :
(i) List the factors on which the magnitude of force experienced by conductor depends.
(ii) When is the magnitude of this force maximum?

25. The electronic configurations of two elements A and B are 2, 8, 1 and 2, 8, 7 respectively.
Which one of them is a metal and which is a non-metal?

Prepared by: M. S. KumarSwamy, TGT(Maths) Page - 4 -


26. How does chemical coordination take place in animals?

SECTION – C
Questions 27 to 33 carry 3 marks each.
27. How do germ cells make a single set of genes from two normal copies of genes?

28. 1000 cm3 of propane is burnt completely in oxygen.


(a) What is the full structural formula of propane?
(b) Draw the electronic structure of propane.
(c) Write the balanced chemical equation for the complete combustion of propane.

29. A yellow brittle element on burning produces a suffocating gas. The solution of this gas in water
turns blue litmus paper red. Answer the following questions :
(a) Is the solution of gas in water acidic or basic?
(b) Is the yellow substance a metal or a non-metal?
(c) Name the gas and the yellow brittle element.

30. Write about power of accommodation of human eye. Explain why the image distance in the eye
does not change when we change the distance of an object from the eye?

31. Write three points of difference between respiration in plants and respiration in animals.
OR
Explain giving any three reasons the significance of transpiration in plants.

32. Design an activity to demonstrate that a bar magnet has a magnetic field around it.

33. (a) What happens to the zygote after fertilization in flowering plants?
(b) What is germination?
(c) Identify the labelled parts A, B and C of the given figure.

SECTION – D
Questions 34 to 36 carry 5 marks each.
34. The diagram given below shows the apparatus that can be used for electrolysis of water. This is
done by passing electricity through acidified water.

Prepared by: M. S. KumarSwamy, TGT(Maths) Page - 5 -


(a) Name the gas A.
(b) Name the gas B.
(c) What type of reaction is taking place? Give a reason for your answer.
(d) Write the chemical equation for this reaction.
(e) Give one test to detect the gas produced at cathode.
OR
(i) A green salt on heating decomposes to produce a colourless suffocating gas and leaves
behind a reddish brown residue. Name the salt and write the decomposition reaction.

(ii) Old oil paintings get blackened over the years due to reactions with pollutant gases in the
air.
(a) Name the gas responsible for discolouration of the paintings.
(b) Give the formula of the black substance formed.
(c) How can the original colour of paintings be restored?
35. (a) A concave mirror of focal length 10 cm can produce a magnified real as well as virtual
image of an object placed in front of it. Draw ray diagrams to justify this statement.
(b) An object is placed perpendicular to the principal axis of a convex mirror of focal length 10
cm. The distance of the object from the pole of the mirror is 10 cm. Find the position of the
image formed.
OR
The image of a candle flame placed at a distance of 30 cm from a mirror is formed on a screen
placed in front of the mirror at a distance of 60 cm from its pole. What is the nature of the
mirror? Find its focal length. If the height of the flame is 2.4 cm, find the height of its image.
State whether the image formed is erect or inverted.

36. (a) Draw a well labelled diagram of stomata.


(b) Explain the structure of stomata. Write the function of guard cells.
OR
(a) What are the functions of stomach?
(b) What are the constituents of gastric juice? Write one function of each.

SECTION – E(Case Study Based Questions)


Questions 37 to 39 carry 4 marks each.
37. Case Study – 1
Some harmful non-biodegradable chemicals enter the bodies of organism through the food chain
and go on concentrating at each trophic level. This phenomenon is called bio-magnification or
biological magnification. It is simply the increase in concentration of a substance in a food
chain, not an organism. Persistent organic pollutants (POPs) are compound that biomagnifies.
Example is of DDT (Pesticide) mercury, Zinc, Lead, Cadmium and other persistent organic
pollutants.
Prepared by: M. S. KumarSwamy, TGT(Maths) Page - 6 -
The concentration effect occurs because these pollutants are metabolised and excreted much
more slowly than the nutrients that are passed from one trophic level to the next. Therefore, they
accumulate in the body (especially in the fat as pollutants have more affinity towards tal). The
accumulation of insecticides (pollutant) Increase try 10 times. Example: If the goat gets
partimillion of DDT from the grasses, it might have 10 ppm in its tissue. The man eating the
goat, will have 100 pm. If the food chain is still greater, the accumulation would still be higher
(Bioamplification). The most common example of blomagnification is of mercury. Large
quantity of mercury is used throughout the world in industries and it is estimated that more than
half of this quantity finds its way two natural ecosystems.

(a) Why maximum concentration of DDT is found in the human beings?


(b) Refer to the given food chain.
Phytoplankton → Zooplankton → Small fish → Large fish → Fish eating birds
If concentration of DDT in small fish is estimated to be 0.5 ppm, then what will be amount of
DDT in zooplankton and large fish?
OR
Explain how pesticides and heavy metals enter the food chain.

38. Case Study – 2


A series of organic compounds having same functional group, with similar or almost identical
chemical characteristics in which all the members can be represented by the same general
formula and the two consecutive members of the series differ by -CH2 group or 14 mass unit in
their molecular formulae is called a homologous series. For example, all the members of alcohol
family can be represented by the general formula, CnH2n+2OH where, n may have the values 1,
2, 3, ... etc. The various members of a particular homologous series are called homologues. The
physical properties such as density, melting point, boiling point, solubility, etc. of the members
of a homologous series show almost regular variation in ascending or descending the series.
(a) What is the general formula of the homologous series of alkynes?
(b) The table shows the formulae of three organic compounds that belong to the same
homologous series.
First member of the homologous series CH3 – O – CH3
Second member of the homologous series CH3CH2 – O – CH3
Third member of the homologous series CH3CH2CH2 – O – CH3
What is the general formula of this series?
(c) Write down the molecular formula of the initial four members of CnH2n+2 homologous series.
OR
Write down the molecular formula of the initial four members of alcohol homologous series.
Prepared by: M. S. KumarSwamy, TGT(Maths) Page - 7 -
39. Case Study - 3
Andre Marie Ampere suggested that a magnet must exert an equal and opposite force on a
current carrying conductor, which was experimentally found to be true. But we know that
current is due to changes in motion. Thus, it is clear that a charge moving in a magnetic field
experience a force, except when it is moving in a direction parallel to it. If the direction of
motion is perpendicular to the direction of magnetic field, the magnitude of force experienced
depends on the charge, velocity (v), strength of magnetic field (B), and sine of the angle
between v and B. Direction of magnetic force is given by Fleming's left hand rule.

(a) If an electron is travelling horizontally towards east, what is the direction of force
experienced by the electron for a magnetic field in vertically downward direction exerts a force
on the electron along?
(b) If a charged particle is moving along a magnetic field line, then what is the magnetic force
on the particle?
(c) What is magnetic lines of force?
OR
A uniform magnetic field exists in the plane of paper pointing from left to right as shown in
figure. In the field an electron and a proton move as shown. Find the forces experienced by the
electron and the proton.

Prepared by: M. S. KumarSwamy, TGT(Maths) Page - 8 -


PM SHRI KENDRIYA VIDYALAYA GACHIBOWLI, GPRA CAMPUS, HYD-32
SAMPLE PAPER TEST 03 FOR BOARD EXAM 2024

SUBJECT: SCIENCE MAX. MARKS: 80


CLASS : X DURATION: 3 HRS
General Instruction:
1. This Question Paper has 5 Sections A-E.
2. Section A has 20 MCQs carrying 1 mark each.
3. Section B has 5 questions carrying 02 marks each.
4. Section C has 6 questions carrying 03 marks each.
5. Section D has 4 questions carrying 05 marks each.
6. Section E has 3 case based integrated units of assessment (04 marks each) with sub-parts of the
values of 1, 1 and 2 marks each respectively.
7. All Questions are compulsory. However, an internal choice in 2 Qs of 5 marks, 2 Qs of 3 marks
and 2 Questions of 2 marks has been provided. An internal choice has been provided in the
2marks questions of Section E
8. Draw neat figures wherever required. Take π =22/7 wherever required if not stated.
SECTION – A
Questions 1 to 20 carry 1 mark each.
1. Oral contraceptives prevent
(a) fertilisation (b) ovulation (c) entrance of sperms in vagina (d) all of these.
2. Complete the given analogy. Non-biodegradable: ________:: Biodegradable: Livestock wastes
(a) Cotton (b) Grass (c) Glass (d) Orange peel
3. The angle between an incident ray and the plane mirror is 35°. The total angle between the
incident ray and reflected ray will be
(a) 35° (b) 70° (c) 105° (d) 110°
4. What is the direction of magnetic field at a point A above the wire
carrying current I as shown in the adjoining figure?
(a) out of the page (b) into the page (c) up the page (d) down the page
5. Which is the correct sequence of air passage during inhalation?
(a) Nostrils → larynx → pharynx → trachea → lungs
(b) Nasal passage → trachea → pharynx → larynx → alveoli
(c) Larynx → nostrils → pharynx → lungs
(d) Nostrils → pharynx → larynx → trachea → alveoli
6. Refer to the given figure depicting locations of several endocrine glands in a human and answer
the following questions.

Prepared by: M. S. KumarSwamy, TGT(Maths) Page - 1 -


Deficiency of hormone of which gland causes:
(i) Dwarfism? (ii) Diabetes mellitus? (iii) Goitre?
(i) (ii) (iii)
(a) p q r
(b) s v r
(c) r u t
(d) q s u
7. A ray of light falls normally or perpendicularly on the surface of a mirror. Which of the
following is not true regarding this?
(a) The angle of incidence for such a ray of light is zero.
(b) The angle of reflection for such a ray of light is zero.
(c) Such ray of light reflects back along the same path.
(d) A ray of light in this case does not follow law of reflection.
8. Among Na, Al, Zn and Cu,
(a) Al is more reactive than Na (b) Zn is more reactive than Al
(c) Zn is more reactive than Cu (d) Al is less reactive than Cu.
9. A particle contains 26 protons, 30 neutrons and 24 electrons. Which statement is true about this
particle?
(a) It is an atom of a noble gas. (b) It is an atom of a non-metal.
(c) It is a negative ion. (d) It is a positive ion.
10. Two wires of same metal have the same length but their cross-sectional area in the ratio 3: 1.
They are joined in series. The resistance of the thicker wire is 10 W. The total resistance of the
combination will be
(a) 40Ω (b) 40/3 Ω (c) 5/2 Ω (d) 100 Ω
11. Which of the following molecules has all its atoms joined together by double covalent bonds?
(a) Methane (b) Water (c) Carbon dioxide (d) Nitrogen trichloride
12. Quick lime combines vigorously with water to form (A) which reacts slowly with the carbon
dioxide in air to form (B). What are (A) and (B)?
(A) (B)
(a) Calcium carbonate Calcium hydroxide
(b) Calcium hydroxide Calcium carbonate
(c) Calcium nitrate Calcium bicarbonate
(d) Calcium bicarbonate Calcium nitrate
13. Observe the given diagram and identify the correct statements.

(i) At anode, oxygen gas is evolved.


(ii) In the test tube covering the anode, the amount of gas collected is double than that of the gas
collected in the test tube covering the cathode.
(iii) At cathode, hydrogen gas is evolved.
(iv) It is a decomposition reaction.
(a) (i), (ii) and (iii) (b) (i), (iii) and (iv) (c) (iii) and (iv) (d) All the statements are correct.

Prepared by: M. S. KumarSwamy, TGT(Maths) Page - 2 -


14. What can be deduced from the following diagram showing a current flowing in a straight wire
surrounded by four compasses?

(a) The magnetic field caused by the four compasses put close together will always cause the
compass needles to point in a circle.
(b) Compass needles are affected by the material of the wire.
(c) Compass needles are affected by the circular electric field in the wire.
(d) Compass needles are affected by the circular magnetic field produced by the current in the
wire.

15. Select the correct representation of the safest method to detect hydrogen gas produced in a
reaction.

16. A metal carbonate X on treatment with a mineral acid liberates a gas which when passed
through aqueous solution of a substance Y gives back X. The substance Y on reaction with the
gas obtained at anode during electrolysis of brine gives a compound Z, which can decolourise
coloured fabrics. The compounds X, Y and Z respectively are
(a) CaCO3, Ca(OH)2, CaOC12
(b) Ca(OH)2, CaO, CaOC12
(c) CaCO3, CaOC12, Ca(OH)2
(d) Ca(OH)2, CaCO3, CaOC12

DIRECTION: In the question number 17 and 20, a statement of Assertion (A) is followed by a
statement of Reason (R).
Choose the correct option
(a) Both assertion (A) and reason (R) are true and reason (R) is the correct explanation of
assertion (A)
(b) Both assertion (A) and reason (R) are true and reason (R) is not the correct explanation of
assertion (A)
(c) Assertion (A) is true but reason (R) is false.
(d) Assertion (A) is false but reason (R) is true.
17. Assertion (A): AlCl3 is a basic salt.
Reason (R): AlCl3 is a salt of strong acid and a weak base.

18. Assertion (A): The connecting wires are made of copper.


Reason (R): The electrical conductivity of copper is high.
Prepared by: M. S. KumarSwamy, TGT(Maths) Page - 3 -
19. Assertion (A): The plants store some of the waste products in their body parts.
Reason (R): Plants can get rid off excess water by transpiration.

20. Assertion (A): Rate of breathing is slower in aquatic organisms than terrestrial organisms.
Reason (R): Amount of dissolved oxygen is low as compared to amount of atmospheric air.

SECTION – B
Questions 21 to 26 carry 2 marks each.
21. (a) What is a gene?
(b) What is heredity?
OR
List the seven pairs of contrasting characters of experimental plant studied by Mendel.

22. Why do stars appear to twinkle ? Explain.

23. Predict (a) the functional group and (b) the number of carbon atoms in a molecule of butene.

24. (a) Name the poles P, Q, R and S of the magnets in the following figures I and II.
(b) State the inference drawn about the direction of the magnetic field lines on the basis of these
diagrams.

OR
What is Maxwell’s cork-screw rule? For what purpose is it used?

25. Write the main functions of the following:


(a) Sensory neuron (b) Cranium

26. List two preparations shown every month by the uterus in anticipation of pregnancy in humans.

SECTION – C
Questions 27 to 33 carry 3 marks each.
27. Differentiate between self-pollination and cross-pollination.

28. How do proteins control the expression of characters? Explain it by taking an example of
tallness in plants as a characteristic.

29. What are magnetic field lines? Justify the following statements:
(i) Two magnetic field lines never intersect each other.
(ii) Magnetic field are closed curves.

30. Identify the substance that is oxidized and the substance that is reduced in the following
reactions:
(a) ZnO(s) + C(s) → Zn(s) + CO(g)
(b) 4Na(s) + O2(g) → 2Na2O(s)
(c) CuO(s) + H2(g) → Cu(s) + H2O(l)

31. Trace the sequence of events which occur when a bright light is focused on your eyes.

Prepared by: M. S. KumarSwamy, TGT(Maths) Page - 4 -


32. A student takes three beakers A, B and C filled with aqueous solutions of glucose, alcohol and
hydrochloric acid respectively as shown in the following figures :

(a) State your observation in terms of glowing of bulb when the switch is 'ON'.
(b) Justify your observations by giving reason in each case.
(c) Mention the change noticed with appropriate reason if the content of beaker B is replaced by
sodium hydroxide solution.
33. (a) "Veins are thin walled and have valves". Justify the statement.
(b) In birds and mammals the left and right side of the heart are separated. Give reason.
OR
What are the necessary conditions for photosynthesis? How is it significant to mankind?

SECTION – D
Questions 34 to 36 carry 5 marks each.
34. Equal lengths of magnesium ribbons are taken in two test tubes (A) and (B). H2SO4 is added to
test tube (A) and H2CO3 in test tube (B) in equal amounts.
(a) Identify the test tube which will show vigorous reaction.
(b) Give reason to support your answer.
(c) Name the gas liberated in both the test tubes. How will you prove its liberation?
(d) Write chemical equations for both the reactions.
(e) Out of the two acids taken above, which one will have lower pH value and lower H+ ion
concentration respectively?
OR
(a) What will you observe when
(i) Methyl orange is added to dilute hydrochloric acid
(ii) A drop of phenolphthalein is added to the solution of lime water?
(b) What is an acid-base indicator? Give two examples of synthetic acid-base indicators.
(c) What are olfactory indicators? Name two substances which can be used as olfactory
indicators?
(d) Solutions X, Y and Z have pH values 8, 10 and 12 respectively. Arrange them in increasing
order of basic strength.
35. (a) To construct a ray diagram we use two rays which are so chosen that it is easy to know their
directions after reflection from the mirror. Use these two rays and draw ray diagram to locate
the image of an object placed between pole and focus of a concave mirror.
(b) A concave mirror produces four times magnified image on a screen. If the objects placed 30
cm in front of the mirror, how far is the screen from the object?
OR
A student focussed the image of a candle flame on a white screen using a convex lens. He noted
down the position of the candle screen and the lens as under
Position of candle = 12.0 cm, Position of convex lens = 50.0 cm,
Position of the screen = 88.0 cm
(i) What is the focal length of the convex lens?
(ii) Where will the image be formed if he shifts the candle towards the lens at a position of 31.0
cm?
(iii) What will be the nature of the image formed if he further shifts the candle towards the lens?
(iv) Draw a ray diagram to show the formation of the image in case (iii) as said above.
Prepared by: M. S. KumarSwamy, TGT(Maths) Page - 5 -
36. (a) What are two different ways in which glucose is oxidised to provide energy in various
organisms?
(b) Write any two differences between the two ways of oxidation of glucose in organisms.
OR
(a) Draw a diagram of excretory system in human beings and label on it : aorta, vena cava,
urinary bladder and urethra.
(b) Write in brief the function of following : (i) Ureter (ii) Urethra (iii) Urinary bladder (iv)
Kidney

SECTION – E(Case Study Based Questions)


Questions 37 to 39 carry 4 marks each.
37. Case Study – 1
The introduction of disposable plastic cups was hailed as a step forward for reasons of hygiene.
These cups made up of clay are the most commonly found solid wastes and cause
environmental pollution.

(a) Kulhads are not recommended over plastic paper disposable cups. Why?
(b) What would be the advantages of disposable paper cups over disposable plastic cups?
OR
The committee members of Aryan's society placed two bins-green coloured and blue coloured
in their premises for garbage collection. Given is the list of few solid wastes generated in his
society. Segregate the wastes in their respective bins.
Paper cup, credit card, fruit and vegetables peels, cardboard, metal rod, aluminium foil, plastic
key chain, pencil, glass sheet

38. Case Study – 2


Non-metals are highly electronegative in nature. They have a tendency to gain electrons in their
valence shell to achieve nearest noble gas configuration. Thus, they form anions and act as good
oxidizing agents.
X + ne    Xn 
(non - metal atom) (anion)

Prepared by: M. S. KumarSwamy, TGT(Maths) Page - 6 -


They react with air or oxygen on heating to form oxides which react with water to form acids.
Thus, nonmetal oxides are acidic in nature. Non-metals do not react with dilute acids at all. This
is because they are electronegative and therefore, cannot displace hydrogen from acids but they
form covalent hydrides when heated with hydrogen.

(a) Name the acid formed when Sulphur trioxide reacts with water.
(b) An element ‘X’ forms an oxide XO2, which is a very useful gas for the process of
photosynthesis. Name the element ‘X’.
(c) Name any two non-metals which form covalent hydrides.
OR
An element Y gains two electrons. What will it form, cation or anion? Write its formula.

39. Case Study - 3


The electrical energy consumed by an electrical appliance is given by the product of its power
rating and the time for which it is used. The SI unit of electrical energy is Joule. Actually, Joule
represents a very small quantity of energy and therefore it is inconvenient to use where a large
quantity of energy is involved. So, for commercial purposes we use a bigger unit of electrical
energy which is called kilowatt hour. 1 kilowatt-hour is equal to 3.6 × 106 joules of electrical
energy.
(a) The electrical refrigerator rated 400 W operates 8 hours a day. The cost of electrical energy
is Rs. 5 per kWh. Find the cost of running the refrigerator for one day?
(b) Calculate the energy transformed by a 5 A current flowing through a resistor of 2 Ω for 30
minutes?
(c) State the difference between kilowatt and kilowatt hour.
OR
Converts the 250 kWh into energy?

Prepared by: M. S. KumarSwamy, TGT(Maths) Page - 7 -


PM SHRI KENDRIYA VIDYALAYA GACHIBOWLI, GPRA CAMPUS, HYD-32
SAMPLE PAPER TEST 04 FOR BOARD EXAM 2024

SUBJECT: SCIENCE MAX. MARKS: 80


CLASS : X DURATION: 3 HRS
General Instruction:
1. This Question Paper has 5 Sections A-E.
2. Section A has 20 MCQs carrying 1 mark each.
3. Section B has 6 questions carrying 02 marks each.
4. Section C has 7 questions carrying 03 marks each.
5. Section D has 3 questions carrying 05 marks each.
6. Section E has 3 case based integrated units of assessment (04 marks each) with sub-parts of the
values of 1, 1 and 2 marks each respectively.
7. All Questions are compulsory. However, an internal choice in 2 Qs of 5 marks, 2 Qs of 3 marks
and 2 Questions of 2 marks has been provided. An internal choice has been provided in the
2marks questions of Section E
8. Draw neat figures wherever required. Take π =22/7 wherever required if not stated.
SECTION – A
Questions 1 to 20 carry 1 mark each.
1. Ayush is putting H2SO4 in the test tube containing water whereas Piyush is putting water in the
test tube containing H2SO4. Which of the two is likely to face danger?
(a) Piyush (b) Ayush (c) None of the two (d) Both Ayush and Piyush
2. Which of the following chemical reactions is incorrect?
Oxygen ( Kreb ' s cycle )
(a) Pyruvate 
in mitochondria
 6CO2  6 H 2 O  38 ATP
in absence of oxygen
(b) Pyruvate  in yeast
2 Lactic acid  2 ATP
in absence of oxygen
(c) Pyruvate  in yeast
2 C2 H 5OH  2CO2  2 ATP
in absence of oxygen
(d) Pyruvate 
Muscle Tissue in animals
 2 Lactic acid  2 ATP

3. In a neuron, conversion of electrical signal to a chemical signal occurs at/in


(a) cell body (b) axonal end (c) dendritic end (d) axon

4. CH3 —CH2 — OH CH3 —COOH


In the above given reaction, alkaline KMnO4 acts as
(a) reducing agent (b) oxidising agent (c) catalyst (d) dehydrating agent
5. The correct sequence of reproductive stages seen in flowering plants is :
(a) Gametes, zygote, embryo, seedling (b) Zygote, gametes, embryo, seedling
(c) Seedling, embryo, zygote, gametes (d) Gametes, embryo, zygote, seedling
6. Which of the following a balanced equation for the following chemical reaction:

Prepared by: M. S. KumarSwamy, TGT(Maths) Page - 1 -


(a) Zn + Ag(NO3)2 → Zn(NO3)2 + 2Ag
(b) 2Zn + 2AgNO3 → Zn(NO3)2 + 2Ag
(c) Zn + 2AgNO3 → Zn(NO3)2 + 2Ag
(d) Zn + AgNO3 → Zn(NO3)2 + Ag

7. Generally, non-metals are not conductors of electricity. Which of the following is a good
conductor of electricity?
(a) Diamond (b) Graphite (c) Sulphur (d) Fullerene

8. Structural formulae of X and Y, which have the same molecular formula, C3H60 are :
(a) X = CH3CH2CHO, Y= CH3COCH3
(b) X = CH3OCH2CH3, Y= CH3COCH3
(c) X = CH3COCH3, Y = CH3COOH
(d) X= CH3CH2OCH3, Y = CH3CH2COOH
9. Read the following statements about asexual reproduction and select the correct ones.
(i) It involves a single parent.
(ii) It is slower than sexual reproduction.
(iii) It produces progeny that are genetically identical with the parent but not with one another.
(iv) The progeny of asexual reproduction can be termed as clones.
(a) (i) and (ii) (b) (ii) and (iii)
(c) (i) and (iv) (d) (i), (iii) and (iv)

10. Two current-conducting wires are hung on a plastic rod. A large current is passed through the
two wires in the direction shown. Which of the following options is correct regarding this?
(I) I and III parts of wire repel each other. (II) II and IV parts of wire repel each other.
(III) I and IV parts of wire repel each other. (IV) II and III parts of wire repel each other.
(a) Only (i) and (ii) (b) Only (i) and (iii) (c) Only (iii) and (iv) (d) Only (i) and (iv)

11. A pea plant is represented by Rr. This represents the


(a) Genetic composition of an individual
(b) Characteristics which are visible in an organism.
(c) Alternate form of genes
(d) None of these

12. Consider these indices of refraction: glass: 1.52; air: 1.0003; water: 1.333. Based on the
refractive indices of three materials, arrange the speed of light through them in decreasing order.
(a) The speed of light in water > the speed of light in air > the speed of light in glass.
(b) The speed of light in glass > the speed of light in water > the speed of light in air.
(c) The speed of light in air > the speed of light in water > the speed of light in glass.
(d) The speed of light in glass > the speed of light in air > the speed of light in water.

13. An electron enters a magnetic field at right angles to it as shown in fig.


The direction of the force acting on the electron will be:

(a) to the right (b) to the left (c) out of the page (d) into the page

Prepared by: M. S. KumarSwamy, TGT(Maths) Page - 2 -


14. The voltage-current (V-I) graph of a metallic conductor at two different temperatures T1 and T2
is shown:

At which temperature is the resistance higher?


(a) T2 (b) T1 (c) Equal at both T2 and T1 (d) Cannot say

15. Which of the following oxide(s) is/are soluble in water to form alkalies?
(i) Na2O (ii) SO2 (iii) K2O (iv) NO2
(a) (i) and (iii) (b) (i) only (c) (ii) and (iv) (d) (iii) only

16. Vinay observed that the stain of curry on a white shirt becomes reddish-brown when soap is
scrubbed on it, but it turns yellow again when the shirt is washed with plenty of water. What
might be the reason for his observation?
(i) Soap is acidic in nature.
(ii) Soap is basic in nature.
(iii) Turmeric is a natural indicator which gives reddish tinge in bases.
(iv) Turmeric is a natural indicator which gives reddish tinge in acids.
(a) (i) and (ii) (b) (ii) and (iii) (c) (i) and (iv) (d) (ii) and (iv)

DIRECTION: In the question number 17 and 20, a statement of Assertion (A) is followed by a
statement of Reason (R).
Choose the correct option
(a) Both assertion (A) and reason (R) are true and reason (R) is the correct explanation of
assertion (A)
(b) Both assertion (A) and reason (R) are true and reason (R) is not the correct explanation of
assertion (A)
(c) Assertion (A) is true but reason (R) is false.
(d) Assertion (A) is false but reason (R) is true.
17. Assertion: Photosynthesis is an anabolic process.
Reason: The process of photosynthesis occurs in chlorophyll.

18. Assertion (a): In a series circuit, the current is constant throughout the electric circuit.
Reason (R): All electric devices need equal currents to operate properly.

19. Assertion (A): Fresh milk in which baking soda is added, takes a longer time to set as curd.
Reason (R): Baking soda decreases the pH value of fresh milk to below 6.

20. Assertion (A): Colonies of yeast multiply in sugar solution.


Reason (R): Sugar is made of sucrose which provides energy for sustaining all life activities.

SECTION – B
Questions 21 to 26 carry 2 marks each.
21. In birds and mammals, the left and right side of the heart are separated. Give reasons.
22. What is electrical resistivity? In a series electrical circuit comprising of a resistor having a
metallic wire, the ammeter reads 5 A. The reading of the ammeter decreases to half when the
length of the wire is doubled. Why?
OR

Prepared by: M. S. KumarSwamy, TGT(Maths) Page - 3 -


Why does an electric bulb become dim when an electric heater in parallel circuit is switched on?
Why does dimness decrease after sometime?

23. Why do veins have thin walls as compared to arteries?

24. Neha did not want to have a child. So, she went to a doctor who suggested her to adopt
contraceptive methods. What are the reasons for adopting contraceptive methods?
OR
In a germinating seed, which parts are known as future shoot and future root? Mention the
function of cotyledon.

25. 3 mL of ethanol is taken in a test tube and warmed gently in a water bath. A 5% solution of
alkaline potassium permanganate is added first drop by drop to this solution, then in excess.
(a) How is 5% solution of KMnO4 prepared?
(b) State the role of alkaline potassium permanganate in this reaction. What happens on adding
it in excess?

26. In a cross between plants with purple flowers and white flowers, the F1 had all white flowers.
When F1 generation was self bred, the F2 generation gave rise to 100 individuals, 75 of which
had white flowers. Make a cross and answer.
(a) What are the genotypes of F2 individual?
(b) What is the ratio of purple flowered plants in F2 generation?

SECTION – C
Questions 27 to 33 carry 3 marks each.
27. Draw ray diagrams showing the image formation by a convex mirror when an object is placed:
(a) at infinity (b) at finite distance from the mirror.

28. When do we consider a student sitting in the class to be myopic? List two causes of this defect.

29. Why are bacteria and fungi called decomposers? List any two advantages of decomposers to the
environment.

30. Derive an expression for electric energy consumed in a device in terms of V, I and t, where V is
the potential difference applied to it, I is the current drawn by it and t is the time for which the
current flows ?

31. Mention any three-information given by a chemical equation.


OR
State the law of conservation of mass as applicable in a chemical reaction. Illustrate with an
example.

32. Which compounds are called (a) alkanes, (b) alkenes and (c) alkynes ? C4H10 belongs to which
of these? Draw two structural isomers of this compound.

33. (i) Plants do not have any nervous system but yet, if we touch a sensitive plant, some observable
changes take place in its leaves. Explain how could this plant respond to the external stimuli and
how it is communicated.
(ii) Name the hormone that needs to be administered to
(a) increase the height of a dwarf plant.
(b) cause rapid cell division in fruits and seeds.
OR
(a) What is endocrine gland?
(b) Name any two endocrine glands present in a human body and write hormones secreted by
them.

Prepared by: M. S. KumarSwamy, TGT(Maths) Page - 4 -


SECTION – D
Questions 34 to 36 carry 5 marks each.
34. (a) An organic compound X with a molecular formula C undergoes oxidation in presence of
alkaline KMnO4 to form a compound Y. X on heating in presence of conc. H2SO4 at 443 K
gives Z, which on reaction gives back 'X'. Identify X, Y and Z and write the reactions involved.
(b) With hard water, "A" compound functions well. The production of shampoos and other
cleaning products uses it. A causes water pollution and isn’t entirely biodegradable. Hard water
makes 'B' less effective. There is no water pollution because it is 100 percent biodegradable.
Identify ‘A’ and ‘B’.
(c) A cyclic compound 'X' has molecular formula. It is unsaturated and burns with sooty flame.
Identify 'X' and write its structural formula. Will it decolorize bromine water or not and why?
(d) An organic compound 'A' is a constituent of antifreeze and has the molecular formula
C2H6O. Upon reaction with alkaline KMNO4 the compound 'A' is oxidized to another 'B’
identify the compound A' and 'B'. Write the chemical equation for the reaction which leads to
the formulation of 'B'.

35. (a) Why is nutrition necessary for the human body?


(b) What causes movement of food inside the alimentary canal?
(c) Why is small intestine in herbivores longer than in carnivores?
(d) What will happen if mucus is not secreted by the gastric glands?
OR
(i) Describe aerobic respiration.
(ii) Describe the process of anaerobic respiration.

36. (i) To construct a ray diagram we use two rays which are so chosen that it is easy to know their
directions after reflection from the mirror. List two such rays and state the path of these rays
after reflection in case of concave mirrors. Use these two rays and draw ray diagram to locate
the image of an object placed between pole and focus of a concave mirror.
(ii) A concave mirror produces three times magnified image on a screen. If the object is placed
20 cm in front of the mirror, how far is the screen from the object?
OR
Rishi went to a palmist to show his palm. The palmist used a special lens for this purpose.
(i) State the nature of the lens and reason for its use.
(ii) Where should the palmist place/hold the lens so as to have a real and magnified image of an
object?
(iii) If the focal length of this lens is 10 cm and the lens is held at a distance of 5 cm from the
palm, use lens formula to find the position and size of the image.
SECTION – E(Case Study Based Questions)
Questions 37 to 39 carry 4 marks each.
37. Case Study – 1
Compounds containing oxygen are of great interest in the field of chemistry. Because of
oxygen's high reactivity, it is most often found in compounds. Oxygen reacts rapidly with group
1 elements. All alkali metal oxides form basic solutions when dissolved in water. The principal
combustion product is the most stable product with respect to the reactants. For example, with
careful control of oxygen, the oxide M2O (where M represents any alkali metal) can be formed
with any of the alkali metals.
When a metal reacts with oxygen to form a metal oxide, a redox reaction occurs.
Metal + Oxygen → Metal oxide
Different metals burn in oxygen with different rates depending on their differing activeness.
The more reactive metal towards oxygen, the brighter and faster the combustion of the metal.
Given figure below shows the experiment is conducted to build the reactivity series of metals.
Oxygen that is used in combustion of other metals is provided by heating solid potassium
manganate (VII). When heated, lithium, sodium, potassium, rubidium, and cesium ignite
Prepared by: M. S. KumarSwamy, TGT(Maths) Page - 5 -
through combustion reactions with oxygen. Glass wool Metal filing Heat up Heat up Potassium
Manganate (VII) crystal

(a) Which metals do not react with oxygen even at high temperature?
(b) What happens when copper metal is heated in air?
(c) Almost all metals combine with oxygen to form metal oxides. Is this statement true?
OR
(c) Which metal oxides are soluble in water? Explain.

38. Case Study – 2


When magnet is brought into the field of another magnet, the field interacts with each pole of
the magnet and each of these poles experience magnetic force. The space surrounding a magnet
where a magnetic force is experienced is called magnetic field.

A magnetic field line is a continuous curve in a magnetic field such that the tangent at any point
on it gives the direction of magnetic field at that point.
(i) Magnetic field is produced by the flow of current in a straight wire. Who discovered this
phenomenon and what is the SI unit of magnetic field? [1]
(ii) Where is the magnetism minimum in a bar magnet? [1]
(iii) Meena draws magnetic field lines of field close to the axis of a current-carrying circular
loop. As she moves away from the centre of the circular loop she observes that the lines keep on
diverging. How will you explain her observation? [2]
OR
(iii) How can it be proved that a magnetic field exists around a current carrying metallic wire?
[2]

39. Case Study – 3


The excretory system is responsible for the elimination of wastes produced by homeostasis.
There are several parts of the body that are involved in this process, such as sweat glands, the
liver, the lungs and the kidney system. Every human has two kidneys. The given diagram
represents the structure of human excretory system. Study the diagram and answer the following
questions.
Prepared by: M. S. KumarSwamy, TGT(Maths) Page - 6 -
(a) What is the role of part 1 in excretion? [1]
(b) Name the structural and functional part of 2? [1]
(c) What will happen, if one kidney of a person is removed? [2]
OR
The urge to urinate can be controlled. Give reason. [2]

Prepared by: M. S. KumarSwamy, TGT(Maths) Page - 7 -


PM SHRI KENDRIYA VIDYALAYA GACHIBOWLI, GPRA CAMPUS, HYD-32
SAMPLE PAPER TEST 05 FOR BOARD EXAM 2024

SUBJECT: SCIENCE MAX. MARKS: 80


CLASS : X DURATION: 3 HRS
General Instruction:
1. This Question Paper has 5 Sections A-E.
2. Section A has 20 MCQs carrying 1 mark each.
3. Section B has 6 questions carrying 02 marks each.
4. Section C has 7 questions carrying 03 marks each.
5. Section D has 3 questions carrying 05 marks each.
6. Section E has 3 case based integrated units of assessment (04 marks each) with sub-parts of the
values of 1, 1 and 2 marks each respectively.
7. All Questions are compulsory. However, an internal choice in 2 Qs of 5 marks, 2 Qs of 3 marks
and 2 Questions of 2 marks has been provided. An internal choice has been provided in the
2marks questions of Section E
8. Draw neat figures wherever required. Take π =22/7 wherever required if not stated.
SECTION – A
Questions 1 to 20 carry 1 mark each.
1. Mohan was confused and did not know how to draw the correct figure for geotropism. So, he
has drawn three figures, as shown below.

Which appears more accurate and why?


(a) (I), (II) (b) (I) only (c) (II), (III) (d) (II) only

2. The diagram shows the reaction between metal and dilute acid.

What is the reason for different behaviour of Mg in test tube B?


(a) Mg is lighter element than dil. HCl
(b) Mg reacts with dil. HCl to produce H2 gas which helps in floating
(c) Mg reacts with dil. HCl to produce N2 gas which helps in floating
(d) Mg reacts with dil. HCl to produce CO2 gas which helps in floating

3. Which of the following will turn phenolphthalein pink?


(a) NaOH(aq) (b) HCl(aq) (c) CH3COOH(aq) (d) H2O

4. While studying the saponification reaction, what do you observe when you mix an equal amount
of colourless vegetable oil and 20% aqueous solution of NaOH in a beaker?
(a) The colour of the mixture has become dark brown.
(b) A brisk effervescence is taking place in the beaker.
Prepared by: M. S. KumarSwamy, TGT(Maths) Page - 1 -
(c) The outer surface of the beaker has become hot.
(d) The outer surface of the beaker has become cold.

5. Why do we store silver chloride in dark-coloured bottles?


(a) To prevent precipitation of silver chloride
(b) To prevent decomposition of silver chloride
(c) To promote decomposition of silver chloride
(d) All of these
6. When sodium hydrogen carbonate is added to ethanoic acid, a gas is evolved. Consider the
following statements about the gas evolved.
(i) It turns lime water milky.
(ii) It is evolved with a brisk effervescence.
(iii) It has a smell of burning sulphur.
(iv) It is also a by-product of respiration.
The correct statements are:
(a) (i) and (ii) only (b) (i) and (iv) only (c) (i), (iii) and (iv) (d) (i), (ii) and (iv)

7. Identify X, Y and Z based on the basis of given information. X is a non-metal, which is an


important constituent of our food. It forms two oxides Y and Z. Y is toxic and it causes
suffocation and sometimes death. Z is responsible for global warming.
(a) X= C, Y= CO, Z=CO2 (b) X = S, Y= SO2, Z= SO3
(c) X=P, Y=P2O3, Z=P2O3 (d) X= O, Y = O2, Z = O3
8. The hetero atoms present in CH3—CH2—O—CH2—CH2—Cl are
(i) Oxygen (ii) Carbon (iii) Hydrogen (iv) Chlorine
(a) (i) and (ii) (b) (ii) and (iii) (c) (iii) and (iv) (d) (i) and (iv)

9. In humans, the life processes are controlled and regulated by:


(a) reproductive and endocrine system (b) respiratory and nervous system
(c) endocrine and digestive system (d) nervous and endocrine system

10. Four students plotted the sketch of the patterns of magnetic field lines representing the magnetic
field around a current carrying straight wire as shown in figures P, Q, R and S. Which one of the
following sketches is correct?

(a) P (b) Q (c) R (d) S

Prepared by: M. S. KumarSwamy, TGT(Maths) Page - 2 -


11. Choose the correct statement about heart.
(a) The upper two chambers of human heart are called ventricles.
(b) The lower two chambers of human heart are called atrium.
(c) The chambers of the heart are separated by a partition called pericardium.
(d) The pulmonary artery carries deoxygenated blood from the right ventricle to the lungs.
12. Offspring formed by asexual method of reproduction have greater similarity among themselves
because
(i) asexual reproduction involves only one parent
(ii) asexual reproduction does not involve gametes
(iii) asexual reproduction occurs before sexual reproduction
(iv) asexual reproduction occurs after sexual reproduction
(a) (i) and (ii) (b) (i) and (iii)
(c) (ii) and (iv) (d) (iii) and (iv)

13. In peas, a pure tall plant (TT) is crossed with a short plant (tt). The ratio of pure tall plants to
short plants in F2 is
(a) 1 : 3 (b) 3 : 1 (c) 1 : 1 (d) 2 : 1

14. What is the effective resistance between points P and Q in the circuit shown below?

(a) 0.15 Ω (b) 0.66 Ω (c) 1.5 Ω (d) 1.75 Ω

15. As per Michael Faraday, the forefinger, middle finger and thumb indicate the direction of:
(a) magnetic field, force and current respectively.
(b) magnetic field, current and force respectively.
(c) current, force and magnetic field respectively.
(d) force, magnetic field and current respectively.
16. In the given diagram, when the magnet is pushed into the solenoid, the pointer of the
galvanometer deflects slightly to the left.

Which of the following would produce a deflection of the pointer towards the right?
(a) Move the solenoid towards the magnet.
(b) Move the solenoid away from the magnet.
(c) Move the magnet faster into the solenoid.
(d) By placing the magnet above the coil.

DIRECTION: In the question number 17 and 20, a statement of Assertion (A) is followed by a
statement of Reason (R).
Choose the correct option
Prepared by: M. S. KumarSwamy, TGT(Maths) Page - 3 -
(a) Both assertion (A) and reason (R) are true and reason (R) is the correct explanation of
assertion (A)
(b) Both assertion (A) and reason (R) are true and reason (R) is not the correct explanation of
assertion (A)
(c) Assertion (A) is true but reason (R) is false.
(d) Assertion (A) is false but reason (R) is true.

17. Assertion: The chemical name of bleaching powder is calcium oxychloride.


Reason: Bleaching powder is used as an oxidising agent in chemical industries.

18. Assertion (a): The opening and closing of the pore is a function of the guard cells.
Reason (R): Stomatal pores are the site for exchange of gases by diffusion.

19. Assertion (A): Lipase help in emulsification of fats.


Reason (R): Lipase hydrolyses fats and oils.

20. Assertion: Strength of an electromagnet can be increased by increasing the number of turns per
unit length in solenoid coil.
Reason: Strength of an electromagnet can be increased by increasing the current flowing
through the solenoid.

SECTION – B
Questions 21 to 26 carry 2 marks each.

21. Why does carbon become stable after sharing four electrons? What type of bond is formed by
sharing?
OR
Why are covalent compounds being poor conductors of electricity? Why do covalent
compounds have low melting and boiling points?
22. Observe the given figure: What happens when the tube is heated?

(a) Write a balanced chemical equation of the reaction.


(b) Identify the brown gas X evolved.
23. Define an ecosystem. Draw a block diagram to show the flow of energy in an ecosystem.

24. What changes are observed in the uterus subsequent to implantation of young embryo?

25. Define the term power of accommodation. Write the modification in the curvature of the eye
lens which enables us to see the nearby objects clearly? Give relation between power and focal
length.
OR
Why is the sun visible to us 2 minutes before actual sunrise and 2 minutes after actual sunset?

26. DDT was sprayed in a lake to regulate breeding of mosquitoes. How would it affect the trophic
levels in the following food chain associated with a lake? Justify your answer.

Prepared by: M. S. KumarSwamy, TGT(Maths) Page - 4 -


SECTION – C
Questions 27 to 33 carry 3 marks each.
27. Based on the group valency of elements, write the molecular formula of the following
compounds giving justification for each:
(a) Oxides of first group elements.
(b) Halides of the elements of group 13.
(c) Compounds formed when an element A of group 2 combines with an element B of group 17.

28. Can two people with brown eyes have a blue-eyed baby? Explain.

29. How is copper extracted from its sulphide ore ? Explain the various steps supported by chemical
equations. Draw labelled diagram for the electrolytic refining of copper.

OR
Write the balanced chemical equations for the following reactions and identify the type of
reaction in each case.
(i) Thermit reaction, iron (III) oxide reacts with aluminium and gives molten iron and
aluminium oxide.
(ii) Magnesium ribbon is burnt in an atmosphere of nitrogen gas to form solid magnesium
nitride.
(iii) Chlorine gas is passed in an aqueous potassium iodide solution to form potassium chloride
solution and solid iodine.
(iv) Ethanol is burnt in air to form carbon dioxide, water and releases heat.

30. A piece of wire having resistance ‘R’ is cut into four equal parts.
(a) How does the resistance of each part compare with the original resistance?
(b) If the four parts are placed in parallel, how will be the resistance of the combination compare
with the resistance of the original wire?

31. A student holding a mirror in his hand, directed the reflecting surface of the mirror towards the
Sun. He then directed the reflected light on to a sheet of paper held close to the mirror.
(a) What should he do to burn the paper?
(b) Which type of mirror does he have?
(c) Will he be able to determine the approximate value of focal length of this mirror from this
activity? Give reason and draw ray diagram to justify your answer in this case.

32. (a) Determine whether the P and Q-marked poles in the following diagram represent the North
or South pole. Why did you choose that response?

(b) Imagine that you are sitting in a chamber with your back to one wall. An electron beam,
moving horizontally from back wall towards the front wall, is deflected by a strong magnetic
field to your right side. What is the direction of magnetic field?
OR

Prepared by: M. S. KumarSwamy, TGT(Maths) Page - 5 -


State the rule to determine the direction of a (a) magnetic field produced around a straight
conductor-carrying current and (b) force experienced by a current-carrying straight conductor
placed in a magnetic field which is perpendicular to it.

33. (a) How does food chain differ from a food web?
(b) Make food chains in (i) forest (ii) pond.

SECTION – D
Questions 34 to 36 carry 5 marks each.
34. If, in Figure Rl = 10 Ohms, R2 = 40 Ohms, R, = 30 Ohms, R4 = 20 Ohms, RA = 60 Ohms and a
12 volt battery is connected to the arrangement, calculate: (a) the total resistance and (b) the
total current flowing in the circuit.

35. What is a homologous series of carbon compounds? List its any two characteristics. Write the
name and formula of the next higher homologous of HCOOH.

36. What is sexual reproduction? Explain how this mode of reproduction gives rise to more viable
variations than asexual reproduction. How does this affect the evolution?
OR
(a) What are dominant and recessive traits?
(b) "Is it possible that a trait is inherited but may not be expressed in the next generation?" Give
a suitable example to justify this statement.

SECTION – E(Case Study Based Questions)


Questions 37 to 39 carry 4 marks each.
37. Case Study - 1
Sohan went door to door posing as a goldsmith. He promised to bring back the glitter of old and
dull gold ornaments. An unsuspecting lady gave a set of gold bangles to him, which he dipped
in a particular solution. The bangles sparkled like new but their weight was reduced drastically.

The lady was sad but after a futile argument, the man beat a hasty retreat.
(a) What is used for dissociation of gold? [1]
(b) Why the weight of the bangle was reduced drastically? [1]
(c) Aqua-regia is a strong oxidising agent. (True or False). [2]

Prepared by: M. S. KumarSwamy, TGT(Maths) Page - 6 -


OR
(c) What is the other name of the solution (in Latin) used by that man posing as the goldsmith?
Why is it so called?

38. Case Study – 2


The growing size of the human population is a cause of concern for all people. The rate of birth
and death in a given population will determine its size. The human population growth year wise
is shown in the below figure. Reproduction is the process by which organisms increase their
population. The process of sexual maturation for reproduction is gradual and takes place while
general body growth is still going on. Some degree of sexual maturation does not necessarily
mean that the mind or body is ready for sexual act or for having and bringing up children.
Various contraceptive devices are being used by human beings to control the size of population.

(a) List two common signs of sexual maturation in boys and girls.
(b) What is the result of reckless female foeticide?
(c) Which contraceptive method changes the hormonal balance of the body? Give an example of it
OR
Write two factors that determine the size of a population.

39. Case Study - 3


Dispersion of light occurs when white light is separated into its different constituent colors
because of refraction and Snell's law.
From Snell's law it can be seen that the angle of refraction of light in a prism depends on the
refractive index of the prism material.

Color l(nm) Freq. (Hz)


Red 760–647 4.3 × 1014
Orange 647-585 4.3 × 1014
Yellow 585-575 5.2 × 1014
Green 575-491 5.6 × 1014
Blue 491-424 6.6 × 1014

Prepared by: M. S. KumarSwamy, TGT(Maths) Page - 7 -


Since the refractive index varies with wavelength, the angle that the light is refracted by will
also vary with wavelength, causing an angular separation of the colors known as angular
dispersion.
For visible light, refraction indices n of most transparent materials (e.g., air, glasses) decrease
with increasing wavelength l:
Colour Wavelength Crown Flint
(nm) glass glass
Violet 396.9 1.533 1.663
Blue 486.1 1.523 1.639
Yellow 589.3 1.517 1.627
Red 656.3 1.515 1.622

Most often seen in recently made puddles on the sides of roads, the oil refracts light much the
same way a rainbow does. Simply put, the thin layer of oil floating on top of the water refracts
the light which then bounces back up off the water underneath, splitting the light rays creating a
pool of rainbow colours.
(a) Which ray is least deviated by a prism?
(b) Which colour of light which has the minimum velocity in the glass prism?
(c) Which optical phenomenon is involved in formation of rainbow?
OR
(c) What is the angle of deviation (d) of a prism?

Prepared by: M. S. KumarSwamy, TGT(Maths) Page - 8 -


PM SHRI KENDRIYA VIDYALAYA GACHIBOWLI, GPRA CAMPUS, HYD-32
SAMPLE PAPER TEST 06 FOR BOARD EXAM 2024

SUBJECT: SCIENCE MAX. MARKS : 80


CLASS : X DURATION : 3 HRS
General Instruction:
1. This question paper consists of 39 questions in 5 sections.
2. All questions are compulsory. However, an internal choice is provided in some questions. A
student is expected to attempt only one of these questions.
3. Section A consists of 20 objective type questions carrying 1 mark each.
4. Section B consists of 6 Very Short questions carrying 02 marks each. Answers to these questions
should in the range of 30 to 50 words.
5. Section C consists of 7 Short Answer type questions carrying 03 marks each. Answers to these
questions should in the range of 50 to 80 words
6. Section D consists of 3 Long Answer type questions carrying 05 marks each. Answer to these
questions should be in the range of 80 to 120 words.
7. Section E consists of 3 source-based/case-based units of assessment of 04 marks each with sub-
parts.

SECTION – A
Questions 1 to 20 carry 1 mark each.
1. The diagram below shows a leaf that was covered by a piece of black paper for a period of 3
days. After 3 days, the paper was removed. On testing, it was found that the area under the black
paper tested negative for starch and the rest tested positive for starch. What was the experiment
trying to test?

(a) If plants make their own food


(b) If light is required for plants to make food
(c) If plants can respire in the absence of light
(d) If plants can survive even in the absence of light

2. What is the difference in the molecular mass of any two adjacent homologues?
(a) 14 amu (b) 15 amu (c) 16 amu (d) 17 amu
Ans: (a) 14 amu
3. What is the focal length of a plane mirror?
(a) Infinity (b) Zero (c) 1 (d) + 1

4. The diagram shows the reaction between metal and dil. acid. What is the reason for different
behaviour of Mg in test tube B?
Prepared by: M. S. KumarSwamy, TGT(Maths) Page - 1 -
(a) Mg is lighter element than dil. HCI.
(b) Mg reacts with dil. HC1 to produce H2 gas which helps in floating.
(c) Mg reacts with dil. HC1 to produce N2 gas which helps in floating.
(d) Mg reacts with dil. HCI to produce CO2 gas which helps in floating.
5. A current-carrying conductor is held as shown:

In which direction should current be passed in the conductor in order to produce a clockwise
magnetic field around the conductor?
(a) North to South (b) South to North (c) East to West (d) West to East
6. For a current in a long straight solenoid N-pole and S-pole are created at the two ends. Among
the following statements, the incorrect statement is:
(a) The field lines inside the solenoid are in the form of straight lines which indicates that the
magnetic field is the same at all points inside the solenoid.
(b) The strong magnetic field produced inside the solenoid can be used to magnetise a piece of
magnetic material like soft iron, when placed inside the coil.
(c) The pattern of the magnetic field associated with the solenoid is different from the pattern of
the magnetic field around a bar magnet.
(d) The N-pole and S-pole exchange position when the direction of current through the solenoid
is reversed.
7. At the time of short circuit, the electric current in the circuit:
(a) vary continuously (b) does not change (c) reduces substantially (d) increases heavily
8. In a study it was found that fused ear lobes were found in more numbers within a population
rather than free ear lobes. What can you infer from the above observation with respect to
dominant/ recessive trait?
(a) Fused ear lobes – dominant (b) Free ear lobes – dominant
(c) Fused ear lobes – recessive (d) Both are dominant
9. The graph below shows the variation of force acting on a conductor with current:

Prepared by: M. S. KumarSwamy, TGT(Maths) Page - 2 -


After analyzing the graph, a student noted the following. Select the correct statement:
(a) The force acting on a conductor increases exponentially with increase in current.
(b) The force acting on a conductor decreases exponentially with increase in current.
(c) The force acting on a conductor increases linearly with increase in current.
(d) The force acting on a conductor decreases linearly with increase in current.

10. What is the minimum resistance which can be made using the following resistors?

(a) 1 Ω (b) 2 Ω (c) 4 Ω (d) 3 Ω

11. Manish’s mother was baking cake in the kitchen. When Manish came back from school, he
detected smell of hot cake from the drawing room. Why?
(a) Due to the presence of olfactory receptors in forebrain
(b) Due to the presence of taste buds
(c) Due to the presence of olfactory receptors in midbrain
(d) Due to the presence of olfactory receptors in hindbrain

12. Which of the following is not the role of decomposers in the ecosystem?
(a) They clean the environment.
(b) They decompose non-biodegradable substances.
(c) They participate in food chain.
(d) They replenish the nutrients in the soil.

13. In the given diagram, when the magnet is pushed into the solenoid, the pointer of the
galvanometer deflects slightly to the left. Which of the following changes would cause the
pointer to deflect through a larger angle?

(a) Move the magnet faster. (b) Move the magnet away from the solenoid.
(c) Unwind some of the turns of the solenoid. (d) Keep the magnet stationary.

14. A straight wire is placed between two poles of a magnet as shown in figure. If an alternating
current passing through a wire then wire will

(a) Move into the page only (b) Move out of the page only
(c) Move out and into the page (d) Remain stationary

15. In the given reaction : ZnO + C → Zn + CO.


I. ZnO is being oxidised.
II. CO is being reduced.
Prepared by: M. S. KumarSwamy, TGT(Maths) Page - 3 -
III. C is being oxidised.
IV. ZnO is being reduced.
Choose the correct statement.
(a) I and II only (b) III and IV only (c) I, II, and III only (d) All of these

16. Common salt besides being used in kitchen can also be used as the raw material for making :
(i) washing soda (ii) bleaching powder (iii) baking soda (iv) slaked lime
(a) (i) and (ii) (b) (i), (ii) and (iv) (c) (i) and (iii) (d) (i), (iii) and (iv)

Q. no 17 to 20 are Assertion - Reasoning based questions. These consist of two statements –


Assertion (a) and Reason (R). Answer these questions selecting the appropriate option given
below:
(a) Both A and R are true and R is the correct explanation of A
(b) Both A and R are true and R is not the correct explanation of A
(c) A is true but R is false
(d) A is false but R is true

17. Assertion (a): Non–biodegradable substances are those substances which cannot be broken
down into simpler harmless substances in nature.
Reason (R): Non–biodegradable substances can cause air pollution and make the air poisonous
when burnt.

18. Assertion (a): The effect of root pressure in transport of water is more important during
daytime.
Reason (R): Transpiration pull is the major driving force in movement of water during the day.

19. Assertion (A): In Fleming's left hand rule, the direction of magnetic field, force and current are
mutually perpendicular.
Reason (R): Fleming's left hand rule is applied to measure the induced current.

20. Assertion (A): Unisexual flowers have separate male and female flowers.
Reason (R): Cucumber, pumpkin and watermelon are the examples of unisexual flowers.

SECTION – B
Questions 21 to 25 carry 2 marks each.

21. (a) Which plant hormone is present in greater concentration in the areas of rapid cell division?
(b) Give one example of a plant growth promoter and a plant growth inhibitor.

22. (a) What is the unit of current? Express it in terms of charge and time.
(b) The following table gives the value of resistivity of some materials:

Material Resistivity (Ohm-m)


A 44 × 10–6
B 1010 – 1012
C 1.62 × 10–6
D 1015 – 1017

Which material would you suggest to be used in electric heating devices? Give reason for your
choice.
OR
(a) On what factors does the resistance of a conductor depend?
(b) Calculate the resistance of an aluminium cable of length 10 km and diameter 2.0 mm if the
resistivity of aluminium is 2.7 × 10–8 m.

Prepared by: M. S. KumarSwamy, TGT(Maths) Page - 4 -


23. Identify the displacement and the double displacement reaction from the following reactions.
(a) HC1(aq) + NaOH(aq) → NaCI(aq) + H2O(l)
(b) Fe(s) + CuSO4(aq) → FeSO4(aq) + Cu(s)
OR
A teacher provided acetic acid, water, lemon juice, aqueous solution of sodium hydrogen
carbonate and sodium hydroxide to students in the school laboratory to determine the pH values
of these substances using pH papers. One of the students reported the pH values of the given
substances as 3, 12, 4, 8 and 14 respectively. Which one of these values is not correct? Write its
correct value stating the reason.

24. What is a rainbow? Draw a well labelled diagram to show the formation of a rainbow.

25. Give reasons for the following observations:


(a) Covalent compounds are poor conductors of electricity.
(b) Highly reactive metals cannot be obtained from their oxides by heating them with carbon.

26. What are the differences between the transport of materials in xylem and phloem?

SECTION – C
Questions 27 to 33 carry 3 marks each.
27. Sahil took five solutions A, B, C, D and E and tested with universal indicator showed pH as 4,
1, 11, 7 and 9 respectively. Which solution is: (a) Neutral (b) Strongly alkaline (c) Strongly
acidic (d) Weakly acidic (E) Weakly alkaline?
Arrange the pH in increasing order of hydrogen ion concentration.

28. (a) Describe how a squirrel uses its hormonal system to react to a dangerous situation.
(b) How do sensory and motor neurons differ from one another?

29. Identify the acid and base which form sodium hydrogen carbonate. Write chemical equation in
support of your answer. State whether this compound is acidic, basic or neutral. Also, write its
pH value.

30. (a) What is an ecosystem? List its two main components.


(b) ‘The number of trophic levels in a food chain is limited’. Justify the statement.

31. Trace the sequence of events which occur when a bright light is focused on your eyes.
OR
List in tabular form three distinguishing features between autotrophic nutrition and
heterotrophic nutrition.

32. Give reasons for the following:


(a) Ionic compounds have high melting and boiling point
(b) Ionic compounds conduct electricity in molten state
(c) Ionic compounds are solid at room temperature and are somewhat hard.

33. (a) What is a solenoid ?


(b) Draw the pattern of magnetic field lines of (i) a current carrying solenoid and (ii) a bar
magnet.
OR
Suppose your parents have constructed a two room house and you want that in the living room
there should be a provision of one electric bulb, one electric fan, a refrigerator and a plug point
for appliances of power upto 2 kilowatt. Draw a circuit diagram showing electric fuse and
earthing as safety devices.

Prepared by: M. S. KumarSwamy, TGT(Maths) Page - 5 -


SECTION – D
Questions 34 to 36 carry 5 marks each.
34. Name three different glands associated with the structures labelled in digestive system as shown
in figure. Also write their secretions and their functions.

OR
(a) Define excretion.
(b) Name the basic filtration unit present in the kidney.
(c) Draw excretory system in human beings and label the following organs of excretory system
which perform following functions: (i) form urine. (ii) is a long tube which collects urine from
kidney. (iii) store urine until it is passed out.

35. Why are certain compounds called hydrocarbons? Write the general formula for homologous
series of alkanes, alkenes and alkynes and also draw the structure of the first member of each
series. Write the name of the reaction that converts alkenes into alkanes and also write a
chemical equation to show the necessary conditions for the reaction to occur.
OR
(a) Explain why carbon forms covalent bond ? Give two reasons for carbon forming a large
number of compounds.
(b) Explain the formation of ammonia molecule.

36. A student wants to project the image of a candle flame on a screen 60 cm in front of a mirror by
keeping the candle flame at a distance of 15 cm from its pole.
(a) Which type of mirror should the student use?
(b) Find the magnification of the image produced.
(c) Find the distance between the object and its image.
(d) Draw a ray diagram to show the image formation in this case and mark the distance between
the object and its image.
OR
(a) Name the lens which can be used as a magnifying glass. For which position of the object a
convex lens form: (i) a real and inverted image of the same size as that of the object? (ii) a
virtual and erect image? Draw ray diagram to justify your answer in each case.
(b) One half of a convex lens is covered with a black paper. Will this lens produce a complete
image of the object? Draw ray diagram to justify your answer.

SECTION – E (Case Study Based Questions)


Questions 35 to 37 carry 4 marks each.
37. Case Study – 1
Manoj performed an experiment to understand that heat is produced when a few drops of
concentrated sulphuric acid is slowly added into a beaker containing water. For this, he took 10

Prepared by: M. S. KumarSwamy, TGT(Maths) Page - 6 -


mL water in a beaker and added a few drops of concentrated H2SO4 to it. Then, he swirled the
beaker slowly. During the process, a vigourous reaction takes place. It is an exothermic process.

(a) Why is it recommended that the acid should be added to water and not water to the acid?
(b) How will the concentration of hydrogen ions gets affected if an acid is diluted?
(c) What is this process called? Define the process.
OR
If we have hydrochloric acid and acetic acid of equal concentration, which will be a stronger
acid and why?

38. Case Study – 2


A student wants to project the image of a candle flame on the walls of the school laboratory by
using a mirror.
(a) Which type of mirror should he use and why?
(b) At what distance, in terms of focal length of the mirror, should he place the candle flame to
get the magnified image on the wall?
(c) Draw a ray diagram to show the formation of the image in this case.
OR
(d) (i) To get the diminished image of the candle flame, where the object must be placed?
(ii) If the image formed by this mirror is inverted and real, then what will be its magnification?
39. Case Study – 3
Bacteria follow an asexual mode of reproduction, called binary fission. A single bacterium
divides into two daughter cells. These are identical to the parent cell as well as to each other.
Replication of DNA within parent bacterium marks the beginning of the fission. Eventually, cell
elongates to form two daughter cells.
The diagram shows the process of binary fission in bacteria

Prepared by: M. S. KumarSwamy, TGT(Maths) Page - 7 -


The rate and timing of reproduction depend upon the conditions like temperature and
availability of nutrients. When there is a favorable condition, E. coli or Escherichia coli
produces about 2 million bacteria every 7 hours.
(a) (i) What is the process of the division of a cell into several cells during reproduction in
Plasmodium?
(ii) A Planaria worm is cut horizontally in the middle into two halves P and Q such that the part
P contains the whole head of the worm. Another Planaria worm is cut vertically into two halves
R and S in such a way that both the cut pieces R and S contain half head each. Which of the cut
pieces of the two Planaria worms could regenerate to form the complete respective worms?
(b) The rapid spreading of bread would on slices of bread is due to spore formation. Explain
spore formation.
OR
(b) Suppose a bacterium reproduces by binary fission every 20 minutes. The new cells survive
and reproduce at the same rate. The graph below shows how the bacterial population would
grow from a single bacterium. What do you conclude?

Prepared by: M. S. KumarSwamy, TGT(Maths) Page - 8 -


PM SHRI KENDRIYA VIDYALAYA GACHIBOWLI, GPRA CAMPUS, HYD-32
SAMPLE PAPER TEST 07 FOR BOARD EXAM 2024

SUBJECT: SCIENCE MAX. MARKS: 80


CLASS : X DURATION: 3 HRS
General Instruction:
1. This Question Paper has 5 Sections A-E.
2. Section A has 20 MCQs carrying 1 mark each.
3. Section B has 5 questions carrying 02 marks each.
4. Section C has 6 questions carrying 03 marks each.
5. Section D has 4 questions carrying 05 marks each.
6. Section E has 3 case based integrated units of assessment (04 marks each) with sub-parts of the
values of 1, 1 and 2 marks each respectively.
7. All Questions are compulsory. However, an internal choice in 2 Qs of 5 marks, 2 Qs of 3 marks
and 2 Questions of 2 marks has been provided. An internal choice has been provided in the
2marks questions of Section E
8. Draw neat figures wherever required. Take π =22/7 wherever required if not stated.
SECTION – A
Questions 1 to 20 carry 1 mark each.
1. Equal volumes of hydrochloric acid and sodium hydroxide solutions of same concentration are
mixed and the pH of the resulting solution is checked with a pH paper. What would be the
colour obtained? (You may use colour guide given in below Figure)

(a) Red (b) Yellow (c) Yellowish green (d) Blue

2. If the key (K) in the arrangement (figure) is taken out (i.e., circuit is made open) and the
magnetic field lines are drawn over the horizontal plane ABCD, the lines are

(a) concentric circles


(b) elliptical in shapes
(c) straight lines parallel to each other
(d) concentric circles near the point O, but of elliptical shapes as we go away from it

Prepared by: M. S. KumarSwamy, TGT(Maths) Page - 1 -


3. Listed here is the reactivity of certain metals.
Metal Reaction with air Reaction with water Reaction with dilute
acids
Gold Does not oxidise or No reaction No reaction
burn
Sodium Burns vigorously to Violent reaction Violent reaction
form oxide
Zinc Burns to form oxides Reacts on heating Reacts to produce
with water hydrogen
Platinum Does not oxidise or No reaction No reaction
burn
Which of the above metals are likely to be obtained in their pure states from the Earth’s crust?
(a) Gold only (b) Sodium only (c) Gold and platinum (d) Zinc and sodium

4. The image shows a bud developing on a Hydra.

How does the bud develop in the Hydra?


(a) bud develops due to separation of body parts of Hydra
(b) bud develops due to repetitive cell division at a specific site
(c) bud develops due to change in the environmental conditions
(d) develops due to attachment of another Hydra at a specific site

5. A student studies that acetic acid is a saturated compound.


The structure of the compound is shown.

Why is acetic acid classified as a saturated compound?


(a) Because there is a single bond between the carbon atoms.
(b) Because there is a double bond between the carbon and oxygen atoms.
(c) Because there is a single bond between the carbon and hydrogen atoms.
(d) Because there is a single bond between the carbon and hydroxide diatom.

6. Why are ionic compounds hard crystalline solids?


(a) Due to the strong force of attraction between positive ions.
(b) Due to the strong force of attraction between negative ions.
(c) Both (a) and (b)
(d) Due to the strong force of attraction between molecules.

Prepared by: M. S. KumarSwamy, TGT(Maths) Page - 2 -


7. Which among the following statement(s) is (are) true? Exposure of silver chloride to sunlight
for a long duration turns grey due to
(i) the formation of silver by decomposition of silver chloride
(ii) sublimation of silver chloride
(iii) decomposition of chlorine gas from silver chloride
(iv) oxidation of silver chloride
(a) (i) only (b) (i) and (iii) (c) (ii) and (iii) (d) (iv) only

8. In an attempt to demonstrate electrical conductivity through an electrolyte, the following


apparatus (see below Figure) was set up.

Which among the following statement(s) is(are) correct?


(i) Bulb will not glow because electrolyte is not acidic
(ii) Bulb will glow because NaOH is a strong base and furnishes ions for conduction.
(iii) Bulb will not glow because circuit is incomplete
(iv) Bulb will not glow because it depends upon the type of electrolytic solution
(a) (i) and (iii) (b) (ii) and (iv) (c) (ii) only (d) (iv) only

9. A metal carbonate reacts with a solution X which forms a salt, water, and a gas Y. What are X
and Y?
(a) X: sodium hydroxide; Y: carbon dioxide
(b) X: sodium hydroxide; Y: hydrogen
(c) X: hydrochloric acid; Y: carbon dioxide
(d) X: hydrochloric acid; Y: hydrogen

10. A student connected a simple circuit for verifying Ohm’s law.

(I) Reading of an ammeter connected in the circuit becomes half when resistance is doubled.
(II) Reading of an ammeter connected in the circuit becomes double when resistance is halved.
(III) Reading of an ammeter connected in the circuit becomes half when voltage is doubled.
(IV) Reading of an ammeter connected in the circuit becomes half when voltage is halved.
Select the incorrect statement:
(a) Only (I) (b) Only (II) (c) Only (III) (d) Both (II) and (III) (1)

11. In order to reduce electricity consumption at home, what kind of appliance should one
purchase?
(a) one which draws low power (b) one which produces less heat
(c) one which operates at a higher voltage (d) one which draws a high amount of current
Prepared by: M. S. KumarSwamy, TGT(Maths) Page - 3 -
12. Which is the correct sequence of parts in human alimentary canal?
(a) Mouth → stomach → small intestine → oesophagus → large intestine
(b) Mouth → oesophagus → stomach → large intestine → small intestine
(c) Mouth → stomach → oesophagus → small intestine → large intestine
(d) Mouth → oesophagus → stomach → small intestine → large intestine

13. The opening and closing of the stomatal pore depends upon:
(a) oxygen (b) temperature (c) water in guard cells (d) concentration of CO2 in stomata

14. What is the direction of magnetic field at a point A above the wire carrying current I as shown
in figure?

(a) Out of the page (b) Into the page (c) Up the page (d) Down the page

15. Two pea plants one with round green seeds (RRyy) and another with wrinkled yellow (rrYY)
seeds produce F1 progeny that have round yellow (RrYy) seeds. When F1 plants are self-
pollinated, the F2 progeny will have new combination of characters. Choose the new
combination from the following
(i) Round, yellow (ii) Round, green (iii) Wrinkled, yellow (iv) Wrinkled, green
(a) (i) and (ii) (b) (ii) and (iii) (c) (i) and (iv) (d) (i) and (iii)

16. Involuntary actions in the body are controlled by


(a) medulla in fore brain (b) medulla in mid brain
(c) medulla in hind brain (d) medulla in spinal cord

DIRECTION: In the question number 17 and 20, a statement of Assertion (A) is followed by a
statement of Reason (R).
Choose the correct option
(a) Both assertion (A) and reason (R) are true and reason (R) is the correct explanation of
assertion (A)
(b) Both assertion (A) and reason (R) are true and reason (R) is not the correct explanation of
assertion (A)
(c) Assertion (A) is true but reason (R) is false.
(d) Assertion (A) is false but reason (R) is true.
Ans: (b) Both assertion (A) and reason (R) are true and reason (R) is not the correct explanation
of assertion (A)

17. Assertion (A): A geneticist crossed two pea plants and got 50% tall and 50% dwarf in the
progeny.
Reason (R): One plant was heterozygous tall and the other was dwarf.

18. Assertion (A): Alternating Current is used in household supply.


Reason (R): AC electric power can be transmitted over long distances without much loss of
energy.

19. Assertion (A): The reaction MnO2 + 4HCl → MnCl2 + 2H2O + Cl2 is an example of a redox
reaction.
Reason (R): In this reaction, HCl is reduced to Cl2 whereas MnO2 is oxidised to MnCl2.

20. Assertion (A): Carbohydrate digestion mainly takes place in small intestine.
Reason (R): Pancreatic juice contains the enzyme lactase.

Prepared by: M. S. KumarSwamy, TGT(Maths) Page - 4 -


SECTION – B
Questions 21 to 26 carry 2 marks each.

21. Why did Mendel choose the pea plant for his experiments?

22. What is peptic ulcer? How is peptic ulcer caused?


OR
List two different functions performed by pancreas in our body.

23. (a) Identify the organism which causes Kala-azar. How does this organism reproduce?
(b) Draw a diagram showing its reproduction.

24. On what factor does the colour of scattered light depend? Explain with an example.
OR
What would happen if danger lights were blue in colour? Justify your answer.

25. A substance X used for coating iron articles is added to a blue solution of a reddish brown metal
Y, the color of the solution gets discharged Identify X and Y & also the type of reaction.

26. The number of malarial patients in a village increased tremendously when large number of frogs
were exported from the village. What could be the cause for this?

SECTION – C
Questions 27 to 33 carry 3 marks each.
27. In the following schematic diagram for the preparation of hydrogen gas as shown in below
Figure, what would happen if following changes are made?

(a) In place of zinc granules, same amount of zinc dust is taken in the test tube
(b) Instead of dilute sulphuric acid, dilute hydrochloric acid is taken
(c) In place of zinc, copper turnings are taken
(d) Sodium hydroxide is taken in place of dilute sulphuric acid and the tube is heated.

28. (a) What are the functions of kidneys?


(b) Name the filtration units present in kidneys.
(c) Name two substances which are selectively reabsorbed from nephric filtrate into the blood.
OR
Explain the process of breathing in man

29. A student holding a mirror in his hand, directed the reflecting surface of the mirror towards the
Sun. He then directed the reflected light on to a sheet of paper held close to the mirror.
(i) What should he do to burn the paper?

Prepared by: M. S. KumarSwamy, TGT(Maths) Page - 5 -


(ii) Which type of mirror does he have?
(iii) Will he be able to determine the approximate value of focal length of this mirror from this
activity? Give reason and draw ray diagram to justify your answer in this case.

30. ‘‘A lens can form a magnified erect image as well as magnified inverted image of an object
placed in front of it.’’ State the nature of this lens and draw ray diagrams to justify the above
statement. Mark the positions of O, F and 2F in the diagram.

31. Ozone therapy is a controversial alternative medical practice that uses ozone gas to fight
disease. Aditya asked his mother whether this ozone therapy is legal to be used. His mother
explained that in 2019, the Food and Drug Administation (FDA) warned against using this
therapy because there is not enough evidence to conclude that it is safe for medical use. They
say that it has no known useful application in supportive or preventive medicine. It is a toxic
and harmful air pollution on ground level. Why is damage to the ozone layer is a cause for
concern? What steps are being taken to limit this damage?

32. Identify the type of chemical reaction in the following statements and define each of them
(a) Digestion of food in our body
(b) Rusting of iron
(c) Heating of manganese dioxide with aluminium powder

33. Draw the pattern of magnetic field lines around a current carrying solenoid and mark the North
and South poles. What can you say about the magnetic field inside the solenoid? How can a
solenoid be used to form an electromagnet?
OR
(i) Draw the pattern of magnetic field lines due to a magnetic field through and around a current
carrying circular loop.

(ii) Name and state the rule to find out the direction of magnetic field inside and around the
loop.

SECTION – D
Questions 34 to 36 carry 5 marks each.

34. (i) For the combination of resistors shown in the following figure, find the equivalent resistance
between M & N.

Prepared by: M. S. KumarSwamy, TGT(Maths) Page - 6 -


(ii) State Joule’s law of heating.
(iii) Why we need a 5 A fuse for an electric iron which consumes 1 kW power at 220 V?
(iv) Why is it impracticable to connect an electric bulb and an electric heater in series?
35. (i) Write the functions of each of the following parts in a human female reproductive system: (a)
Ovary (b) Uterus (c) Fallopian tube
(ii) Write the structure and functions of placenta in a human female.
OR
(i) “Use of a condom is beneficial for both the sexes involved in a sexual act.” Justify this
statement giving two reasons.
(ii) How do oral contraceptive help in avoiding pregnancies?
(iii) What is sex selective abortion? How does it affect a healthy society? (State any one
consequence)

36. Sahil bought a compound ' X' on electrolysis in aqueous solution produces a strong base Y'
along with two gases 'A' and 'B'. 'B' is used in manufacture of bleaching powder. Identify X, Y,
A and B. Write chemical equations
OR
Raman took a sodium compound 'X', which is also used in soda-acid fire extinguisher, and
heated it gives a sodium compound 'Y' along with water and carbon dioxide. 'Y' on
crystallisation forms a compound 'Z'.
(a) Identify 'X', 'Y' and 'Z'. Write chemical equations of the reactions taking place.
(b) How can we obtain Y from Z? Write equation.
(c) Write any two uses of the compound 'Z'.
SECTION – E(Case Study Based Questions)
Questions 37 to 39 carry 4 marks each.
37. Case Study – 1
In human beings, the sex of the individual is largely genetically determined. When two germ
cells combine, they will restore the normal number of chromosomes in the progeny, ensuring
the stability of the DNA of the species. Nivedita has dark brown eyes, like her mother. But she
has the same shaped nose as her father.
(i) Explain why Nivedita has features from both of her parents.
(ii) Explain why Nivedita is genetically female.
OR
(ii) In humans, the gene for black hair colour is B and gene for brown hair colour is b. What will
be the hair colour of person having the genetic constitution (a) BB (b) bb?

38. Case Study – 2


A spherical mirror is a mirror which has the shape of a piece cut out of a spherical surface.
There are two types of spherical mirrors: concave, and convex. As is well-known, these types of
mirrors magnify objects placed close to them. It has been observed that as rays from a distant
object depart further from the principal axis of a concave mirror they are brought to a focus ever
closer to the mirror, as shown in fig. below: This lack of perfect focusing of a spherical mirror is
called spherical aberration.

Prepared by: M. S. KumarSwamy, TGT(Maths) Page - 7 -


(a) Where is the principal focus of a parabolic mirror?
(b) After reflection from a concave mirror, rays of light parallel. to the principal. axis converge
at a point. What do we call that point?
OR
(b) Define centre of curvature and focal length of a mirror.
39. Case Study – 3
The nature of non-metals is strongly electronegative. To obtain the nearest noble gas
configuration, they frequently add electrons to their valence shell. They become anions as a
result, which makes them effective oxidising agents.
X + ne- → Xn-
(non-metal atom) (anion)
They react with air or oxygen on heating to form oxides which react with water to form acids.
Thus, non-metal oxides are acidic in nature. Non-metals do not react with dilute acids at all.
This is because they are electronegative and therefore, cannot displace hydrogen from acids but
they form covalent hydrides when heated with hydrogen.
(a) Name the acid formed when sulphur trioxide reacts with water.
(b) An element 'X' forms an oxide XO, which is a very useful gas used in the process of
photosynthesis. Identify the element 'X'.
(c) Non-metals generally act as oxidising agents. Justify. Identify an element which produces
basic oxide on reacting with oxygen?
OR
(c) Name three elements which form covalent hydride?

Prepared by: M. S. KumarSwamy, TGT(Maths) Page - 8 -


PM SHRI KENDRIYA VIDYALAYA GACHIBOWLI, GPRA CAMPUS, HYD-32
SAMPLE PAPER TEST 08 FOR BOARD EXAM 2024

SUBJECT: SCIENCE MAX. MARKS: 80


CLASS : X DURATION: 3 HRS
General Instruction:
1. This Question Paper has 5 Sections A-E.
2. Section A has 20 MCQs carrying 1 mark each.
3. Section B has 5 questions carrying 02 marks each.
4. Section C has 6 questions carrying 03 marks each.
5. Section D has 4 questions carrying 05 marks each.
6. Section E has 3 case based integrated units of assessment (04 marks each) with sub-parts of the
values of 1, 1 and 2 marks each respectively.
7. All Questions are compulsory. However, an internal choice in 2 Qs of 5 marks, 2 Qs of 3 marks
and 2 Questions of 2 marks has been provided. An internal choice has been provided in the
2marks questions of Section E
8. Draw neat figures wherever required. Take π =22/7 wherever required if not stated.
SECTION – A
Questions 1 to 20 carry 1 mark each.
1. Study the diagram given alongside and identify the gas formed in the reaction.

(a) Carbon dioxide which extinguishes the burning candle.


(b) Oxygen due to which the candle burns more brightly.
(c) Sulphur dioxide which produces a suffocating smell.
(d) Hydrogen which while burning produces a popping sound.
2. A scientist in a chemistry lab wants to make salt of pH 5.5 using acid and base. The table shows
the acid and base present in the lab.
1 HCl
2 NaOH
3 H2CO3
4 NH4OH
5 CH3COOH
Which of the acid and base he should use for the reaction?
(a) CH3COOH and NaOH (b) HCl and NaOH (c) HCl and NH4OH (d) H2CO3 and NaOH
3. Which of the following are correct structural isomers of butane?

(a) (i) and (iii) (b) (ii) and (iv) (c) (i) and (ii) (d) (iii) and (iv)
Prepared by: M. S. KumarSwamy, TGT(Maths) Page - 1 -
4. Which of the following statements about transmission of nerve impulse is incorrect?
(a) Nerve impulse travels from dendritic end towards axonal end
(b) At the dendritic end electrical impulses bring about the release of some chemicals which
generate an electrical impulse at the axonal end of another neuron
(c) The chemicals released from the axonal end of one neuron cross the synapse and generate a
similar electrical impulse in a dendrite of another neuron
(d) A neuron transmits electrical impulses not only to another neuron but also to muscle and
gland cells

5. A child is standing in front of a magic mirror. She finds the image of her head bigger, the
middle portion of her body of the same size and that of the legs smaller. The following is the
order of combinations for the magic mirror from the top.
(a) Plane, convex and concave (b) Convex, concave and plane
(c) Concave, plane and convex (d) Convex, plane and concave

6. The image shows the transport of gases in body through heart and lungs.

Which option correctly shows the transport of oxygen to the cell?


(a) Lungs → pulmonary vein → left atrium → left ventricle → aorta → body cells.
(b) Lungs → pulmonary artery → right atrium → right ventricle → vena cava → body cells.
(c) Lungs → pulmonary artery → left atrium → left ventricle → vena cava → body cells.
(d) Lungs → pulmonary vein → right atrium → right ventricle → aorta → body cells.

7. A cell, a resistor, a key and ammeter are arranged as shown in circuit diagrams.

The current recorded in the ammeter will be


(a) maximum in (i) (b) maximum in (ii) (c) maximum in (iii) (d) same in all three circuits

8. The path of a ray of light coming from air passing through a rectangular glass slab is traced by
four students shown as A, B, C and D in the figure. Which one of them is correct?

Prepared by: M. S. KumarSwamy, TGT(Maths) Page - 2 -


9. A student was asked to write a stepwise procedure to demonstrate that carbon dioxide is
necessary for photosynthesis. He wrote the following steps. The wrongly worded step is:

(a) Both potted plants are kept in dark room for at least three days.
(b) Bottom of the bell jars is sealed to make them air tight.
(c) Both potted plants are kept in sunlight after the starch test.
(d) A leaf from both the plants is taken to test the presence of starch.

10. Four cells each of emf 1.5V and the internal resistance 0.5 Ω are connected in series but one cell
is wrongly connected as shown in figure.

The net voltage and net internal resistance between A and B is


(a) 6 V, 2 Ω (b) 4.5 V, 1.5 Ω (c) 3V, 1 Ω (d) 3V, 2 Ω

11. Electrical impulse travels in a neuron from


(a) Dendrite → axon → axonal end → cell body
(b) Cell body → dendrite → axon → axonal end
(c) Dendrite → cell body → axon → axonal end
(d) Axonal end → axon → cell body → dendrite

12. Which of the following occurs during oxygen shortage in muscle cells?

(a) Only X (b) Only Y (c) Only Z (d) Any of them - X, Y or Z

Prepared by: M. S. KumarSwamy, TGT(Maths) Page - 3 -


13. Anand took four colourless solutions P, Q, R and S, and performed the following tests. What is
the definite conclusion that Anand can reach?
Solution P Solution Q Solution R Solution S
With methyl No change in Turns red No change in No change in
orange colour colour colour
With No change in No change in No change in Turns pink
phenolphthalein colour colour colour
With red litmus No change in No change in No change in Turns litmus
colour colour colour blue
With blue litmus No change in Turns litmus red No change in No change in
colour colour colour
(a) Both P and S are salt solutions. (b) Both Q and S are basic solutions.
(c) Both Q and R are salt solutions. (d) Both P and R are neutral solutions.

14. When a 4V battery is connected across an unknown resistor there is a current of 100 mA in the
circuit. The value of the resistance of the resistor is:
(a) 4 Ω (b) 40 Ω (c) 400 Ω (d) 0.4 Ω

15. In the given food chain, suppose the amount of energy at the fourth trophic level is 5 kJ, what
will be the energy available at the producer level?
Grass → Grasshopper → Frog → Snake → Hawk
(a) 5 kJ (b) 50 kJ (c) 500 kJ (d) 5000 kJ

16. Disposable plastic plates should not be used because:


(a) they are made of materials with light weight
(b) they are made of toxic materials
(c) they are made of biodegradable materials
(d) they are made of non-biodegradable materials

DIRECTION: In the question number 17 and 20, a statement of Assertion (A) is followed by a
statement of Reason (R).
Choose the correct option
(a) Both assertion (A) and reason (R) are true and reason (R) is the correct explanation of
assertion (A)
(b) Both assertion (A) and reason (R) are true and reason (R) is not the correct explanation of
assertion (A)
(c) Assertion (A) is true but reason (R) is false.
(d) Assertion (A) is false but reason (R) is true.
Ans: (b) Both assertion (A) and reason (R) are true and reason (R) is not the correct explanation
of assertion (A)

17. Assertion (A): A compass needle is placed near a current carrying wire. The deflection of the
compass needle decreases when the magnitude of an electric current in the wire is increased.
Reason (R): Strength of a magnetic field at a point near the conductor increases on increasing
the current.

18. Assertion (A) : A convex mirror is used as a rear view driver's mirror.
Reason (R) : Convex mirrors have a wider field of view as they are curved outwards. They also
give an erect, although diminished image.

19. Assertion(A): The sex of a child in human beings will be determined by the type of
chromosome he/she inherits from the father.
Reason(R): A child who inherits ‘X’ chromosome from his father would be a girl (XX), while a
child who inherits a ‘Y’ chromosome from the father would be a boy (XY).

Prepared by: M. S. KumarSwamy, TGT(Maths) Page - 4 -


20. Assertion (A) : The energy of charged particle moving in a uniform magnetic field does not
change.
Reason (R) : Work done by magnetic field on the charge is zero.

SECTION – B
Questions 21 to 26 carry 2 marks each.

21. 21. Identify the substances oxidised and the substances reduced in the following reactions.
(i) ZnO(s) + C(s) → Zn + CO(g)
(ii) 4Na(s) + O2(g) → 2Na2O(s)
OR
Which among the following changes are exothermic or endothermic in nature?
(i) Decomposition of ferrous sulphate
(ii) Dilution of sulphuric acid
(iii) Dissolution of sodium hydroxide in water
(iv) Dissolution of ammonium chloride in water

22. ‘Variations are beneficial to the species but not necessarily for the individual’. Justify.

23. Draw a ray diagram to explain the term angle of deviation.


OR
Write about power of accommodation of human eye. Explain why the image distance in the eye
does not change when we change the distance of an object from the eye?

24. “All plants give out oxygen during day and carbon dioxide during night”. Do you agree with
this statement? Give reason.

25. Give one example of an organic compound present in biogas. Draw electron dot structure of this
compound.

26. Ibrahim applied sodium hydroxide to the lustrous, divalent element M. In the reaction mixture,
he saw bubbles beginning to form. When hydrochloric acid was used to treat this element, he
recorded the same findings. Tell him how to recognise the gas that was produced. For both
reactions, write chemical equations.

SECTION – C
Questions 27 to 33 carry 3 marks each.
27. Identify the acid and the base from which sodium chloride is obtained. Which type of salt is it?
When is it called rock salt? How is rock salt formed?

28. An element A burns with golden flame in air. It reacts with another element B, atomic number
17 to give a product C. An aqueous solution of product C on electrolysis gives a compound D
and liberates hydrogen. Identify A, B, C and D. Also write down the equations for the reactions
involved.

29. Rakesh performed the experiment to study the dispersion of light by a glass prism and drew the
following figure.
(a) The colour at position marked 3 and 5 are similar to the colour of the sky and the colour of
gold metal respectively. Is the above statement made by the student correct or incorrect? Justify.
(b) Which of the above shown positions correspond approximately to the colour of:
(i) a solution of potassium permanganate?
(ii) danger or stop signal lights?
OR
Refractive indices of media A, B, C and D are given:
Media: A B C D
Refractive Index: 133 144 152 165
Prepared by: M. S. KumarSwamy, TGT(Maths) Page - 5 -
In which of these four media is the speed of light () Minimum and (ji) Maximum?
Give reasons. Find the refractive index of medium A with respect to medium B.

30. A cross was carried out between pure breed tall pea plant with pure dwarf pea plant and F1
progeny was obtained. Later, F1, progeny was selfed to obtain F2 progeny. Answer the following
questions:
(a) What is the phenotype of the F1 progeny and why?
(b) Give the phenotypic ratio of the F2 progeny.
(c) Why is F2 progeny different from the F1 progeny?

31. How do carbohydrates, proteins and fats get digested in human beings?
OR
What are the adaptations of leaf for photosynthesis?

32. You have been selected to talk on “ozone layer and its protection” in the school assembly on
‘Environment Day’.
(a) Why should ozone layer be protected to save the environment?
(b) List any two ways that you would stress in your talk to bring in awareness amongst your
fellow friends that would also help in protection of ozone layer as well as the environment.

33. Study the following circuit and answer the questions that follows:

(a) State the type of combination of two resistors in the circuit.


(b) How much current is flowing through (i) 10 Ohms and (ii) 15 Ohms resistor?
(c) What is the ammeter reading?

SECTION – D
Questions 34 to 36 carry 5 marks each.

34. A metal carbonate X on reacting with an acid gives a gas which when passed through a solution
Y gives the carbonate back. On the other hand, a gas G that is obtained at anode during
electrolysis of brine is passed on dry Y, it gives a compound Z, used for disinfecting drinking
water. Identify X, Y, G and Z.
OR
An organic compound A on heating with concentrated H2SO4 forms a compound B which on
addition of one mole of hydrogen in presence of Ni forms a compound C. One mole of
compound C on combustion forms two moles of CO2 and 3 moles of H2O. Identify the
compounds A, B and C and write the chemical equations of the reactions involved.

35. What is a solenoid? Draw the pattern of magnetic field lines of (i) a current carrying solenoid
and (ii) a bar magnet. List two distinguishing features between the two fields.
OR
With the help of a labelled circuit diagram illustrating the pattern of field lines of the magnetic
field around a current-carrying straight long conducting wire. How is the right-hand thumb rule
useful to find the direction of the magnetic field associated with a current-carrying conductor?

Prepared by: M. S. KumarSwamy, TGT(Maths) Page - 6 -


36. (i) Label the endocrine glands given in figure below:

(ii) A hot object has been touched by you. Draw a diagram that shows the steps that result in a
response, such as a hasty hand pullback.
(iii) How are involuntary actions and reflex actions different from each other?
SECTION – E(Case Study Based Questions)
Questions 37 to 39 carry 4 marks each.
37. Case Study – 1
We know that the characteristics of image formed by a concave mirror depend on the position of
the object with respect to the mirror.
When an object is placed between F and infinity, the image formed is real and inverted. But
when the object is placed between F and mirror it cannot be obtained on the screen. The image
formed in this case is virtual, erect and magnified. Such image may be seen by looking in the
mirror directly.

When the object is moved from focus towards infinity, the image moves from infinity towards
focus and its size decreases.
When object is placed at 2F image of the same size is formed at 2F, itself.
(i) What will be the nature of image if an object is placed 10 cm in front of a concave mirror of
focal length 20 cm?
(ii) What is the minimum distance between the object and its real image for concave mirror?
(iii) A candle flame 3 cm high is placed at a distance of 3 m from a wall. How far from the wall
must a concave mirror be placed in order that it may form an image of the flame 9 cm high on
the wall?
OR

(iii) Draw a ray diagram when an object is placed near a concave mirror at a distance of one-
fourth the radius of curvature of the concave mirror?

Prepared by: M. S. KumarSwamy, TGT(Maths) Page - 7 -


38. Case Study – 2
The ability of carbon atoms to form chains leads to the existence of a series of compounds that
have same functional group (and hence similar chemical properties) and only differ from each
other by the presence of an additional carbon atom and its two associated hydrogen atoms in the
molecule (which causes the physical properties to change in a regular manner). A series of
compounds related in this way is said to form an homologous series.
The point about chemical properties is best illustrated by the sections that follow, on different
homologous series. The changes in physical properties are a result of the changes that occur in
the strength of van der Waals’ forces with increasing molar mass and in some cases a change in
molecular polarity. The simplest illustration of the effect of chain length on physical properties
is the, variation of the boiling point of the alkanes with the number of carbon atoms in the chain,
as illustrated in figure.

(i) All the members of a homologous series have similar chemical properties. Why?
(ii) In the graph shown, which has the higher boiling point and why?
Hydrocarbon with 3 carbon atoms or hydrocarbon with 6 carbon atoms
(iii) (a) What is the boiling point of heptane as shown in the graph?
(b) Why the curve is initially steep and flattens at the end?

OR

(iii) (a) Write the molecular formula of the 2nd and the 3rd member of the homologous series
whose first member is methane.
(b) Write the next homologue of each of the following: I. C2H4 II. C4H6

39. Case Study - 3


Have you ever wondered how water reaches the top of tall trees, or for that matter how and why
substances move from one cell to the other? Plants need to move molecules over very long
distances, much more than animals do; they also do not have a circulatory system in place.
Water taken up by the roots has to reach all parts of the plant, up to the very tip of the growing
stem. The photosynthates or food synthesised by the leaves have also to be moved to all parts
including the root tips embedded deep inside the soil. Movement across short distances, say
within the cell, across the membranes and from cell to cell within the tissue has also to take
place. Over small distances substances move by diffusion and by cytoplasmic streaming
supplemented by active transport. Transport over longer distances proceeds through the vascular
system (the xylem and the phloem) and is called translocation.

Prepared by: M. S. KumarSwamy, TGT(Maths) Page - 8 -


The table below gives the results of an experiment carried out to study the factors affecting the
rate of transpiration:

Amount of water transpired in 1 Hour (ml)


Normal With Fan With Heater With Lamp
Arrowhead 3.6 7.5 6.6 4
Coleus 0.9 6 3.9 3
Devil's Ivy 2.9 4.6 4.1 3
Dieffenbachia 4.1 7.7 6 3.9
English Ivy 1.8 5.1 3.2 2.1
Geranium 1.2 4.7 5.8 2.4
Rubber Plant 4.9 8.4 6.8 4.3
Weeping Fig 3.3 6.1 4.9 2.5
Zebra Plant 4.2 7.6 6.1 3.2

(a) What do we call the absorption and upward movement of minerals from roots to leaves.
(b) Root pressure is involved in transpiration. Is this statement true?
(c) Effect of root pressure in transport of water is more important at night. Justify.
OR
(c) What is the function of phloem?

Prepared by: M. S. KumarSwamy, TGT(Maths) Page - 9 -


PM SHRI KENDRIYA VIDYALAYA GACHIBOWLI, GPRA CAMPUS, HYD-32
SAMPLE PAPER TEST 09 FOR BOARD EXAM 2024

SUBJECT: SCIENCE MAX. MARKS: 80


CLASS : X DURATION: 3 HRS
General Instruction:
1. This Question Paper has 5 Sections A-E.
2. Section A has 20 MCQs carrying 1 mark each.
3. Section B has 5 questions carrying 02 marks each.
4. Section C has 6 questions carrying 03 marks each.
5. Section D has 4 questions carrying 05 marks each.
6. Section E has 3 case based integrated units of assessment (04 marks each) with sub-parts of the
values of 1, 1 and 2 marks each respectively.
7. All Questions are compulsory. However, an internal choice in 2 Qs of 5 marks, 2 Qs of 3 marks
and 2 Questions of 2 marks has been provided. An internal choice has been provided in the
2marks questions of Section E
8. Draw neat figures wherever required. Take π =22/7 wherever required if not stated.
SECTION – A
Questions 1 to 20 carry 1 mark each.
1. Which option illustrates the location of centre that controls the feelings associated with hunger
(M) and the centre that allows a person to walk in a straight line (N)?

2. A sample of soil is mixed with water and allowed to settle. The clear supernatant solution turns
the pH paper yellowish-orange. Which of the following would change the colour of this pH
paper to greenish-blue?
(a) Lemon juice (b) Vinegar (c) Common salt (d) An antacid

3. Generally metals react with acids to give salt and hydrogen gas. Which of the following acids
does not give hydrogen gas on reacting with metals (except Mn and Mg)?
(a) H2SO4 (b) HCl (c) HNO3 (d) All of these

4. A student adds an equal amount of copper sulphate solution in two beakers. He adds zinc in
beaker P and silver in beaker Q. The student observes that the color of the solution in beaker P

Prepared by: M. S. KumarSwamy, TGT(Maths) Page - 1 -


changes while no change is observed in beaker Q. Which option arranges the metals in
increasing order of reactivity?
(a) Copper-silver-zinc (b) Zinc-copper-silver (c) Silver-copper-zinc (d) Silver-zinc-copper

5. Identify gas A in the following experiment.

(a) Nitrogen (b) Hydrogen (c) Oxygen (d) Carbon dioxide

6. What happens when a solution of an acid is mixed with a solution of a base in a test tube?
(i) The temperature of the solution increases
(ii) The temperature of the solution decreases
(iii) The temperature of the solution remains the same
(iv) Salt formation takes place
(a) (i) only (b) (i) and (iii) (c) (ii) and (iii) (d) (i) and (iv)

7. Which of the following reactions is a neutralisation reaction?


(a) 4Na + O2 → 2Na2O
(b) Fe + 2HCl → FeCl2 + H2
(c) MgO + H2O → Mg(OH)2
(d) HNO3 + NaOH → NaNO3 + H2O

8. Identify the unsaturated compounds from the following


(i) Propane (ii) Propene (iii) Propyne (iv) Chloropropane
(a) (i) and (ii) (b) (ii) and (iv) (c) (iii) and (iv) (d) (ii) and (iii)

9. Which of the following statement is incorrect?


(a) For every hormone there is a gene. (b) For every protein there is a gene.
(c) For production of every enzyme there is a gene. (d) For every molecule of fat there is a gene.

10. Select the mismatched pair


(a) Adrenaline: Pituitary gland (b) Testosterone: Testes
(c) Estrogen : Ovary (d) Thyroxin: Thyroid gland

11. A student carries out an experiment and plots the V - I graph of three samples of nichrome wire
with resistances R1, R2 and R3 respectively as shown in figure. Which of the following is true?

(a) R1 = R2 = R3 (b) R1 > R2 > R3 (c) R3 > R2 > R1 (d) R2 > R3 > R1

Prepared by: M. S. KumarSwamy, TGT(Maths) Page - 2 -


12. What is the direction of magnetic field at a point A above the wire carrying current I as shown
in figure?

(a) Out of the page (b) Into the page (c) Up the page (d) Down the page

13. A cylindrical conductor of length l and uniform area of cross section A has resistance R.
Another conductor of length 2l and resistance R of the same material has area of crosssection:
(a) A/2 (b) 3A/2 (c) 2A (d) 3A

14. A uniform magnetic field exists in the plane of paper pointing from left to right as shown in
below Figure. In the field an electron and a proton move as shown. The electron and the proton
experience

(a) forces both pointing into the plane of paper


(b) forces both pointing out of the plane of paper
(c) forces pointing into the plane of paper and out of the plane of paper, respectively
(d) force pointing opposite and along the direction of the uniform magnetic field respectively

15. Which one among the following is not removed as a waste product from the body of a plant?
(a) Resins and Gums (b) Urea (c) Dry Leaves (d) Excess Water

16. If the structure marked X in the diagram given below is blocked, then which of the processes
will not occur?

(a) Transpiration and respiration (b) Transpiration, photosynthesis and respiration


(c) Respiration, transpiration and transportation (d) Respiration and photosynthesis

DIRECTION: In the question number 17 and 20, a statement of Assertion (A) is followed by a
statement of Reason (R).
Choose the correct option
(a) Both assertion (A) and reason (R) are true and reason (R) is the correct explanation of
assertion (A)
(b) Both assertion (A) and reason (R) are true and reason (R) is not the correct explanation of
assertion (A)
(c) Assertion (A) is true but reason (R) is false.
(d) Assertion (A) is false but reason (R) is true.

Prepared by: M. S. KumarSwamy, TGT(Maths) Page - 3 -


17. Assertion (A) : The compass placed near the current-carrying wire remains stationary.
Reason (R) : The current flowing through a wire gives rise to a magnetic field.

18. Assertion (A): Hydrogen gas is not evolved when a metal reacts with nitric acid.
Reason (R): Nitric acid is a strong oxidising agent.

19. Assertion (A) : Acquired traits cannot be passed from one generation to next generation.
Reason (R) : Inaccuracy during DNA copying of acquired trait is minimum.

20. Assertion(A): Spores are formed in sporangia.


Reason(R): Spores grow into separate individuals in moist conditions.

SECTION – B
Questions 21 to 25 carry 2 marks each.

21. Why do fire flies glow at night?


OR
A dilute ferrous sulphate solution was gradually added to the beaker containing acidified
potassium permanganate solution. The light purple colour of the solution fades and finally
disappears. Write the correct explanation for this observation.

22. Answer the following:


(i) Which hormone is responsible for the changes noticed in females at puberty?
(ii) Dwarfism results due to deficiency of which hormone?
(iii) Blood sugar level rises due to deficiency of which hormone?
(iv) Iodine is necessary for the synthesis of which hormone?

23. Give two examples of decomposers. State their important role in nature.

24. List in tabular form three distinguishing features between cerebrum and cerebellum.

25. A student sitting at the back of the classroom cannot read clearly the letters written on the
blackboard. What advice will a doctor give to her? Draw ray diagram for the correction of this
defect.
OR
Why do we see a rainbow in the sky only after rainfall?

26. What will happen if mucus is not secreted by the gastric glands?

SECTION – C
Questions 27 to 33 carry 3 marks each.
27. A chemical compound ‘X’ is used in the soap and glass industry. It is prepared from brine.
(i) Write the chemical name, common name and chemical formula of ‘X’.
(ii) Write the equation involved in its preparation.
(iii) What happens when it is treated with water containing Ca or Mg salts?

28. 1 g of copper powder was taken in a China dish and heated. What change takes place on
heating? When hydrogen gas is passed over this heated substance, a visible change is seen in it.
Give the chemical equations of reactions, the name and the color of the products formed in each
case.
29. State the events occurring during the process of photosynthesis. Is it essential that these steps
take place one after the other immediately?
OR
Bile juice does not have any digestive enzyme but still plays a significant role in the process of
digestion. Justify the statement.
Prepared by: M. S. KumarSwamy, TGT(Maths) Page - 4 -
30. A lens produces a magnification of –0.5. Is this a converging or diverging lens? If the focal
length of the lens is 6 cm, draw a ray diagram showing the image formation in this case.

31. An object 6 cm in size is placed at 50 cm in front of a convex lens of focal length 30 cm. At
what distance from the lens should a screen be placed in order to obtain a sharp image of the
object? Find the nature and size of the image. Also draw labelled ray diagram to show the image
formation in this case.

32. (i) What is an electromagnet? List any two uses.


(ii) Draw a labelled diagram to show how an electromagnet is made.
OR
With the help of a labelled diagram, explain the distribution of magnetic field due to a current
through a circular loop. Why is it that if a current carrying coil has n turns the field produced at
any point is n times as large as that produced by a single turn?
(ii) Draw a pattern of magnetic field formed around a current carrying solenoid. What happens
to the magnetic field when the current through the solenoid is reversed?

33. Plastic cups were used to serve tea in trains in early days—these could be returned to the
vendors, cleaned and reused. Later, Kulhads were used instead of plastic cups. Now, paper cups
are used for serving tea. What are the reasons for the shift from Plastic to Kulhads and then
finally to paper cups?

SECTION – D
Questions 34 to 36 carry 5 marks each.

34. (i) How will you infer with the help of an experiment that the same current flows through every
part of the circuit containing three resistors R1, R2 and R3 in series connected to a battery of V
volts?
(ii) Study the following circuit and find out:

(a) Current in 12  resistor.


(b) Difference in the readings of A1 and A2, if any.
35. Explain Mendel’s experiment with peas on inheritance of characters considering only one
visible contrasting character.
OR
In the following crosses write the characteristics of the progeny.
Cross Progeny
(i) RR YY × RR YY ___________________________
Round, yellow and round, yellow
(ii) Rr Yy × Rr Yy ___________________________
Round, yellow and round, yellow
(iii) rr yy × rr yy ___________________________
Wrinkled, green and wrinkled, green
(iv) RR YY × rr yy ___________________________
Prepared by: M. S. KumarSwamy, TGT(Maths) Page - 5 -
Round, yellow and wrinkled, green

36. What are esters? How are esters prepared? Write the chemical equation for the reaction
involved. What happens when an ester reacts with sodium hydroxide? Write the chemical
equation for the reaction and also state the name and use of this reaction.
OR
A compound C (molecular formula, C2H4O2) reacts with Na-metal to form a compound R and
evolves a gas which burns with a pop sound. Compound C on treatment with an alcohol A in
presence of an acid forms a sweet smelling compound S (molecular formula C3H6O2). On
addition of NaOH to C, it also gives R and water. S on treatment with NaOH solution gives
back R and A.
Identify C, R, A, S and write down the reactions involved.

SECTION – E(Case Study Based Questions)


Questions 37 to 39 carry 4 marks each.
37. Case Study – 1
A student added 10 g of calcium carbonate in a rigid container, secured it tightly and started to
heat it. After some time, an increase in pressure was observed, the pressure reading was then
noted at intervals of 5 mins and plotted against time, in a graph as shown below.

(i) During which interval did maximum decomposition took place?


(ii) Marble statues are corroded or stained when they repeatedly come into contact with polluted
rain water. Identify the main reason.

(iii) What happens when calcium carbonate decompose? What could be done to increase the rate
of decomposition of CaCO3?
OR
(iii) Is decomposition of limestone endothermic? Give reason.

Prepared by: M. S. KumarSwamy, TGT(Maths) Page - 6 -


38. Case Study – 2
Sahil performed an experiment to study the inheritance pattern of genes. He crossed tall pea
plants (TT) with short pea plants (tt) and obtained all tall plants in F1 generation.
(i) What will be set of genes present in the F1 generation?
(ii) Give reason why only tall plants are observed in F1 progeny.
(iii) When F1 plants were self-pollinated, a total of 800 plants were produced. How many of
these would be tall, medium height or short plants? Give the genotype of F2 generation.
OR
(iii) When F1 plants were cross - pollinated with plants having tt genes, a total of 800 plants
were produced. How many of these would be tall, medium height or short plants? Give the
genotype of F2 generation.

39. Case Study – 3


Light spectrum is the many different wavelengths of energy produced by light source. Light is
measured in nanometers (nm). Each nanometer represents a wavelength of light or band of light
energy. Visible light is the part of spectrum from 380 nm to 780 nm.

Isaac Newton was the first to use a glass prism to obtain the spectrum of sunlight. He tried to
split the colours of the spectrum of white light further by using another similar prism. He then
placed a second identical prism in an inverted position with respect to the first prism. This
allowed all the colours of the spectrum to pass through second prism. He found a beam of white
light emerging from the other side of the second prism. This observation gave Newton the idea
that the sunlight is made up of seven colours.
(i) (a) What is the range of wavelength of visible light spectrum?
(b) What do you understand by light spectrum?
(ii) Explain the process of refraction when critical angle between an equilateral prism and air is
45 degree, if the incident ray is perpendicular to the refracting surface?
OR
(ii) (a) Why do different rays deviate differently in the prism?
(b) How will you use two identical prisms so that a narrow beam of white light incident on one
prism emerges out of the second prism as white light?

Prepared by: M. S. KumarSwamy, TGT(Maths) Page - 7 -


PM SHRI KENDRIYA VIDYALAYA GACHIBOWLI, GPRA CAMPUS, HYD-32
SAMPLE PAPER TEST 10 FOR BOARD EXAM 2024

SUBJECT: SCIENCE MAX. MARKS: 80


CLASS : X DURATION: 3 HRS
General Instruction:
1. This Question Paper has 5 Sections A-E.
2. Section A has 20 MCQs carrying 1 mark each.
3. Section B has 5 questions carrying 02 marks each.
4. Section C has 6 questions carrying 03 marks each.
5. Section D has 4 questions carrying 05 marks each.
6. Section E has 3 case based integrated units of assessment (04 marks each) with sub-parts of the
values of 1, 1 and 2 marks each respectively.
7. All Questions are compulsory. However, an internal choice in 2 Qs of 5 marks, 2 Qs of 3 marks
and 2 Questions of 2 marks has been provided. An internal choice has been provided in the
2marks questions of Section E
8. Draw neat figures wherever required. Take π =22/7 wherever required if not stated.
SECTION – A
Questions 1 to 20 carry 1 mark each.
1. Which of the following reactions is a neutralisation reaction?
(a) 4Na + Cl2 → 2Na2O (b) Fe + 2HCl → FeCl2 + H2
(c) MgO + H2O → Mg(OH)2 (d) HNO3 + NaOH → NaNO2 + H2O

2. Identify the unsaturated compounds from the following


(i) Propane(ii) Propene (iii) Propyne (iv) Chloropropane
(a) (i) and (ii) (b) (ii) and (iv) (c) (iii) and (iv) (d) (ii) and (iii)

3. Sodium hydroxide is termed an alkali while Ferric hydroxide is not because:


(a) Sodium hydroxide is a strong base, while Ferric hydroxide is a weak base.
(b) Sodium hydroxide is a base which is soluble in water while Ferric hydroxide is also a base
but it is not soluble in water.
(c) Sodium hydroxide is a strong base while Ferric hydroxide is a strong acid.
(d) Sodium hydroxide and Ferric hydroxide both are strong base but the solubility of Sodium
hydroxide in water is comparatively higher than that of Ferric hydroxide.

4. Which among the following statement(s) is (are) true?


Exposure of silver chloride to sunlight for a long duration turns grey due to:
(i) The formation of silver by decomposition of silver chloride.
(ii) Sublimation of silver chloride.
(iii) Decomposition of chlorine gas from silver chloride.
(iv) Oxidation of silver chloride.
(a) (i) only (b) (i) and (iii) (c) (ii) and (iii) (d) (iv) only

5. When Sodium bicarbonate reacts with dilute hydrochloric acid, the gas evolved is:
(a) Hydrogen; it gives pop sound with burning match stick.
(b) Hydrogen; it turns lime water milky.
(c) Carbon dioxide; it turns lime water milky.
(d) Carbon dioxide; it blows off a burning match stick with a pop sound.

6. A metal ribbon ‘X’ burns in oxygen with a dazzling white flame forming a white ash ‘Y’. The
correct description of X, Y and the type of reaction is:
(a) X = Ca ; Y = CaO ; Type of reaction = Decomposition
(b) X = Mg ; Y = MgO ; Type of reaction = Combination
Prepared by: M. S. KumarSwamy, TGT(Maths) Page - 1 -
(c) X = Al ; Y = Al2O3 ; Type of reaction = Thermal decomposition
(d) X = Zn ; Y = ZnO ; Type of reaction = Endothermic

7. A cross between pea plant with white flowers (vv) and pea plant with violet flowers (VV)
resulted in F2 progeny in which ratio of violet (VV) and white (vv) flowers will be:
(a) 1 : 1 (b) 2 : 1 (c) 3 : 1 (d) 1 : 3

8. In plants the role of cytokinin is:


(a) Promote cell division. (b) Wilting of leaves.
(c) Promote the opening of stomatal pore. (d) Help in the growth of stem.

9. Acid present in tomato is:


(a) Methanoic acid (b) Acetic acid (c) Lactic acid (d) Oxalic acid

10. The figure given below shows a schematic plan of blood circulation in humans with labels (i) to
(iv). Identify the correct label with its functions?

(a) (i) Pulmonary vein - takes impure blood from body part.
(b) (ii) Pulmonary artery - takes blood from lung to heart.
(c) (iii) Aorta - takes blood from heart to body parts.
(d) (iv) Vena cava - takes deoxygenated blood from body parts to right auricle.

11. Length of pollen tube depends on the distance between:


(a) pollen grain and upper surface of stigma
(b) pollen grain on upper surface of stigma and ovule
(c) pollen grain in anther and upper surface of stigma
(d) upper surface of stigma and lower part of style

12. Which among the following statements are true for sexual reproduction in flowering plants?
(i) It requires two types of gametes. (ii) Fertilisation is a compulsory event.
(iii) It always results in formation of zygote. (iv) Offspring formed are clones.
(a) (i) and (iv) (b) (i), (iii) and (iv)
(c) (i), (ii) and (iii) (d) (i), (ii) and (iv)

13. In an electrical circuit, three incandescent bulbs A, B, and C of rating 40 W, 100 W, and 60 W,
respectively are connected in parallel to an electric source. Which of the following is likely to
happen regarding their brightness?
(a) Brightness of all the bulbs will be the same.
(b) Brightness of bulb A will be the maximum.
(c) Brightness of bulb B will be more than that of A and C.
(d) Brightness of bulb C will be less than that of B.
Prepared by: M. S. KumarSwamy, TGT(Maths) Page - 2 -
14. A student learns that magnetic field strength around a bar magnet is different at every point.
Which diagram shows the correct magnetic field lines around a bar magnet?
(a) (b)

(c) (d)

15. The resistance of a resistor is reduced to half of its initial value. If other parameters of the
electrical circuit remain unaltered, the amount of heat produced in the resistor will become:
(a) four times (b) two times (c) half (d) one fourth

16. If the field lines in a magnetic field are parallel and equidistant, the magnetic field is
(a) Uniform (b) Non uniform (c) Zero (d) None of these

DIRECTION: In the question number 17 and 20, a statement of Assertion (A) is followed by a
statement of Reason (R).
Choose the correct option
(a) Both assertion (A) and reason (R) are true and reason (R) is the correct explanation of
assertion (A)
(b) Both assertion (A) and reason (R) are true and reason (R) is not the correct explanation of
assertion (A)
(c) Assertion (A) is true but reason (R) is false.
(d) Assertion (A) is false but reason (R) is true.

17. Assertion (A): Reaction of Quicklime with water is an exothermic reaction.


Reason (R): Quicklime reacts vigorously with water releasing a large amount of heat.

18. Assertion (A): The wires supplying current to an electric heater are not heated appreciably.
Reason (R): Resistance of connecting wires is very small and H<< R.

19. Assertion (A): At puberty, in boys, voice begins to crack and thick hair grows on face.
Reason (R): At puberty, there is decreased secretion of testosterone in boys.

20. Assertion (A): The inner walls of the small intestine have finger like projections called villi
which are rich in blood.
Reason (R): These villi have a large surface area to help the small intestine in completing the
digestion of food.

SECTION – B
Questions 21 to 26 carry 2 marks each.
21. Draw a well labelled diagram of stomata. List two functions of stomata.

Prepared by: M. S. KumarSwamy, TGT(Maths) Page - 3 -


22. Why is KOH solution kept in the test-tube inside the airtight conical flask while doing the
experiment of respiration of seeds?

23. Observe the following diagram and answer the questions following it :

(i) Identify the defect of vision shown.


(ii) List its two causes.
(iii) Name the type of lens used for the correction of this defect.
OR
The colour of clear sky from the earth appears blue but from the space it appears black. Why?

24. A student took a small amount of copper oxide in a conical flask and added dilute hydrochloric
acid to it with constant stirring. He observed a change in colour of the solution.
(i) Write the name of the compound formed and its colour.
(ii) Write a balanced chemical equation for the reaction involved.
OR
The industrial process used for the manufacture of caustic soda involves electrolysis of an
aqueous solution of compound ‘X’. In this process, two gases ‘Y’ and ‘Z’ are liberated. ‘Y’ is
liberated at cathode and ‘Z’, which is liberated at anode, on treatment with dry slaked lime
forms a compound ‘B’. Name X, Y, Z and B.

25. Name the part of brain which is responsible for the following actions :
(i) Maintaining posture and balance
(ii) Beating of heart
(iii) Thinking
(iv) Blood pressure
OR
Where are auxins synthesized in a plant ? Which organ of the plant shows :
(i) Positive phototropism
(ii) Negative geotropism
(iii) Positive hydrotropism

26. While teaching the chapter "Our Environment", the teacher stressed upon the harmful effects of
burning of fossil fuels, plastic paper, etc. The students noticed the extensive use of plastic and
polythene in daily life, which can be avoided and the surroundings can be kept clean. They
decided to make their school "Plastic and Polythene" free and motivated each other for its
minimum use.
(a) Why should the use of polythene and plastic be reduced in daily life?
(b) In what way the students would have avoided the use of plastic and polythene in their
school?

SECTION – C
Questions 27 to 33 carry 3 marks each.
27. Name the hormone which regulates carbohydrate, protein and fat metabolism in our body.
Which gland secretes this hormone? Why is it important for us to have iodised salt in our diet?
OR
Reproduction is one of the most important characteristics of living beings. Give three reasons in
support of the statement.

Prepared by: M. S. KumarSwamy, TGT(Maths) Page - 4 -


28. If the image formed by a lens for all positions of an object placed in front of it is always erect
and diminished, what is the nature of this lens ? Draw a ray diagram to justify your answer. If
the numerical value of the power of this lens is 10D, what is its focal length in the Cartesian
system?
OR
An object of height 10 cm is placed 25 cm away from the optical centre of a converging lens of
focal length 15 cm. Calculate the image-distance and height of the image formed.

29. Refractive index of water with respect to air is 1.33 and that of diamond is 2.42.
(a) In which medium does the light move faster, water or diamond?
(b) What is the refractive index of diamond with respect to water?

30. (i) Why is an alternating current (A.C.) considered to be advantageous over direct current (D.C.)
for the long distance transmission of electric power ?
(ii) How is the type of current used in household supply different from the one given by a
battery of dry cells ?
(iii) How does an electric fuse prevent the electric circuit and the appliances from a possible
damage due to short circuiting or overloading.
OR
For the current carrying solenoid as shown, draw magnetic field lines and give reason to explain
that out of the three points A, B and C, at which point the field strength is maximum and at
which point it is minimum?

31. (a) From the following groups of organisms, create a food chain which is the most advantageous
for Human being in terms of energy.

(b) State the possible disadvantage if the cereal plant is growing in a soil rich in pesticides.
(c) Construct a food web using the organisms mentioned above.

32. (i) While electrolysing water before passing the current some drops of an acid are added. Why ?
Name the gases liberated at cathode and anode. Write the relationship between the volume of
gas collected at anode and the volume of gas collected at cathode.
(ii) What is observed when silver chloride is exposed to sunlight ? Give the type of reaction
involved.

33. (i) Suggest a safe procedure of diluting a strong concentrated acid.


(ii) Name the salt formed when sulphuric acid is added to sodium hydroxide and write its pH.
(iii) Dry HCl gas does not change the colour of dry blue litmus paper. Why?

SECTION – D
Questions 34 to 36 carry 5 marks each.

34. Explain the process of nutrition in Amoeba.

35. (i) How is electric current related to the potential difference across the terminals of a conductor?
Prepared by: M. S. KumarSwamy, TGT(Maths) Page - 5 -
Draw a labelled circuit diagram to verify this relationship.
(ii) Why should an ammeter have low resistance ?
(iii) Two V - I graphs A and B for series and parallel combinations of two resistors are as
shown. Giving reason state which graph shows (a) series, (b) parallel combination of the
resistors.

36. (i) Draw the structure of the following compounds: (a) Butanoic acid (b) Chloropentane
(ii) How are structure (i) and structure (ii) given below related to one another ? Give reason to
justify your answer.

Draw one more possible structure for above case.


(iii) Differentiate between saturated and unsaturated carbon compounds on the basis of their
general formula.
OR

(i) What are soaps? Explain the mechanism of cleansing action of soap with the help of a
labelled diagram.
(ii) Detergents are better than soaps. Justify.

SECTION – E(Case Study Based Questions)


Questions 37 to 39 carry 4 marks each.
37. Case Study – 1
The pH of a solution is a measure of its hydrogen ion (H+) concentration. It is measured
generally using pH scale. The values on pH scale ranges from 0 to 14.
A pH of 1 is very acidic and corresponds to a high concentration of H+ ions. A pH of 14 is very
basic and corresponds to a low concentration of H+ ions. The pH of a neutral solution is 7. The
table given below shows the pH and H+ ion concentration of some common aqueous solutions.
The leftmost column shows the number of moles of H+ ions in 1 mole of liquid.
The pH and Hydrogen ion (H+) Concentration of Some Solutions

Prepared by: M. S. KumarSwamy, TGT(Maths) Page - 6 -


H+
Concentration pH Solution
(moles)
10–1 1
10–2 2 Gastric (stomach) juice, cola, lemon
juice
10–3 3 Vinegar
–4
10 4 Tomato juice
10–5 5 Black coffee, rain water
–6
10 6 Urine
–7
10 7 Pure water
10–8 8 Sea water
–9
10 9 Baking soda
10–10 10
10–11 11 Milk of magnesia
10–12 12 Household bleach
–13
10 13 Oven cleaner
–14
10 14
(i) How is the hydrogen ion concentration and pH related to each other?
(ii) On the basis of above table, arrange the following in the decreasing order of H+
concentration.
Pure water, tomato juice, milk of magnesia, sea water
(iii) A solution of pH 2 is filled in two separate beakers. A few drops of methyl orange and
phenolphthalein are added into separate solutions. How will the colour of the indicators change?
OR
(iii) Two solutions X and Y have pH values of 3.0 and 9.5 respectively. Which of these will turn
litmus solution from blue to red and which will turn phenolphthalein from colourless to pink?

38. Case Study – 2


The ability of a medium to refract light is expressed in terms of its optical density. Optical
density has a definite connotation. It is not the same as mass density. On comparing two media,
the one with the large refractive index is optically denser medium than the other. The other
medium with lower refractive index is optically rarer. Also the speed of light through a given
medium is inversely proportional to its optical density.
(i) Determine the speed of light in diamond if the refractive index of diamond with respect to
vacuum is 2.42. Speed of light in vacuum is 3×108 m/s. [1]
(ii) Refractive indices of glass, water and carbon disulphide are 1.5, 1.33 and 1.62 respectively.
If a ray of light is incident in these media at the same angle (say θ), then write the increasing
order of the angle of refraction in these media. [1]
(iii) The speed of light in glass is 2×108 m/s and in water is 2.25×108 m/s. [2]
(a) Which one of the two is optically denser and why ?
(b) A ray of light is incident normally at the water-glass interface when it enters a thick glass
container filled with water. What will happen to the path of the ray after entering the glass ?
Give reason.
OR
(iii) The absolute refractive indices of water and glass are 4/3 and 3/2 respectively. If the speed
of light in glass is 2×108 m/s, find the speed of light in (i) vacuum and (ii) water. [2]

39. Case Study – 3


The most obvious outcome of the reproductive process is the generation of individuals of
similar design, but in sexual reproduction they may not be exactly alike. The resemblances as
well as differences are marked. The rules of heredity determine the process by which traits and
characteristics are reliably inherited. Many experiments have been done to study the rules of
inheritance.
(i) Why an offspring of human being is not a true copy of his parents in sexual reproduction? [1]

Prepared by: M. S. KumarSwamy, TGT(Maths) Page - 7 -


(ii) While performing experiments on inheritance in plants, what is the difference between F1
and F2 generation? [1]
(iii) Why do we say that variations are useful for the survival of a species over time ? [2]
OR
(iii) Study Mendel’s cross between two plants with a pair of contrasting characters. [2]
RRYY × rryy
Round Yellow Wrinkled Green
He observed 4 types of combinations in F2 generation. Which of these were new combinations ?
Why do new features which are not present in the parents, appear in F2 generation ?

Prepared by: M. S. KumarSwamy, TGT(Maths) Page - 8 -

You might also like